Sei sulla pagina 1di 100

FUNDAMENTA L

18 THEOREMS OF
VE C TOR ANALYSIS
18.1 Greens Theorem (ET Section 17.1)
Preliminary Questions 
1. Which vector field F is being integrated in the line integral x 2 d y e y d x?
  
SOLUTION The line integral can be rewritten as e y d x + x 2 d y. This is the line integral of F = e y , x 2 along
the curve.
2. Draw a domain in the shape of an ellipse and indicate with an arrow the boundary orientation of the boundary curve.
Do the same for the annulus (the region between two concentric circles).
SOLUTION The orientation on C is counterclockwise, meaning that the region enclosed by C lies to the left in traversing
C.
C

For the annulus, the inner boundary is oriented clockwise and the outer boundary is oriented counterclockwise. The
region between the circles lies to the left while traversing each circle.

3. The circulation of a gradient vector field around a closed curve is zero. Is this fact consistent with Greens Theorem?
Explain.
SOLUTION Greens Theorem asserts that
    
Q P
F ds = P dx + Q dy = dA (1)
C C D x y
If F is a gradient vector field, the cross partials are equal, that is,
P Q Q P
= =0 (2)
y x x y

Combining (1) and (2) we obtain C F ds = 0. That is, Greens Theorem implies that the integral of a gradient vector
field around a simple closed curve is zero.

4. Which of the following vector fields possess the following property: For every simple closed curve C, F ds is
C
equal to the area enclosed by C?
(a) F = y, 0
(b) F = x, y

2
(c) F = sin(x 2 ), x + e y
SOLUTION By Greens Theorem,
   
Q P
F ds = dx dy (1)
C D x y
S E C T I O N 18.1 Greens Theorem (ET Section 17.1) 1195

We compute the curl of each one of the given fields.


(a) Here, P = y and Q = 0, hence Q P
x y = 0 (1) = 1. Therefore, by (1),
 
F ds = 1 d x d y = Area(D)
C D

(b) We have P = x and Q = y, therefore Q P


x y = 0 0 = 0. By (1) we get
 
F ds = 0 d x d y = 0  = Area(D)
C D
2
(c) In this vector field we have P = sin(x 2 ) and Q = x + e y . Therefore,
Q P
= 1 0 = 1.
x y
By (1) we obtain
 
F ds = 1 d x d y = Area(D).
C D

Exercises 
1. Verify Greens Theorem for the line integral x y d x + y d y, where C is the unit circle, oriented counterclockwise.
C
SOLUTION
Step 1. Evaluate the line integral. We use the parametrization ( ) = cos , sin , 0 2 of the unit circle. Then

d x = sin d , d y = cos d

and

x y d x + y d y = cos sin ( sin d ) + sin cos d = cos sin2 + sin cos d

The line integral is thus


  2
xy dx + y dy = cos sin2 + sin cos d
C 0
 2  2  
sin3 2 cos 2 2
= cos sin2 d + sin cos d = =0 (1)
0 0 3 0 4 0

D
x

Step 2. Evaluate the double integral. Since P = x y and Q = y, we have


Q P
= 0 x = x
x y
We compute the double integral in Greens Theorem:
    
Q P
dx dy = x d x d y = x dx dy
D x y D D
1196 C H A P T E R 18 F U N D A M E N TA L TH E O R E M S O F V E C T O R A N A LY S I S (ET CHAPTER 17)

The integral of x over the disk D is zero, since by symmetry the positive and negative values of x cancel each other.
Therefore,
  
Q P
dx dy = 0 (2)
D x y
Step 3. Compare. The line integral in (1) is equal to the double integral in (2), as stated in Greens Theorem.


2
2. Let I = F ds, where F = y + sin x 2 , x 2 + e y and C is the circle of radius 4 centered at the origin.
C
(a) Which is easier: evaluating I directly or using Greens Theorem?
(b) Evaluate I using the easier method.
SOLUTION

(a) Using the parametrization ( ) = 4 cos , 4 sin  for the circle, we have

d x = 4 sin d , d y = 4 cos d

and

(y + sin x 2 )d x + (x 2 + e y )d y = 4 sin + sin(16 cos2 ) (4 sin ) d + (16 cos2 + e16 sin ) 4 cos d
2 2


= 16 sin2 4 sin sin(16 cos2 ) + 64 cos3 + 4 cos e16 sin d
2

The line integral is thus


  2
16 sin2 4 sin sin(16 cos2 ) + 64 cos3 + 4 cos e16 sin d
2
F ds = (1)
C 0
2
We examine the double integral in Greens Theorem. Since P = y + sin x 2 and Q = x 2 + e y , we have
Q P
= 2x 1
x y
The double integral is thus
   
Q P
dA = (2x 1) d x d y (2)
D x y D

Clearly, the double integral in (2) is much easier to evaluate than the line integral in (1).
(b) To evaluate the double integral in (2), we notice that by symmetry the integral of 2x over D is zero, since the positive
and negative values of x cancel each other. Hence,
     
Q P
dA = (2x 1) d x d y = 2x d x d y 1 dx dy
D x y D D D

= 0 Area(D) = 42 = 16

In Exercises 311, use Greens Theorem to evaluate the line integral. Orient the curve counterclockwise unless otherwise
indicated.

3. y 2 d x + x 2 d y, where C is the boundary of the unit square 0 x 1, 0 y 1
C
SOLUTION
S E C T I O N 18.1 Greens Theorem (ET Section 17.1) 1197

1 C

x
1
D

We have P = y 2 and Q = x 2 , therefore


Q P
= 2x 2y
x y
Using Greens Theorem we obtain
    
Q P
y2 d x + x 2 d y = dA = (2x 2y) d x d y = 2 x dx dy 2 y dx dy
C D x y D D D
By symmetry, the positive and negative values of x cancel each other in the first integral, so this integral is zero. The
second double integral is zero by similar reasoning. Therefore,

y2 d x + x 2 d y = 0 0 = 0
C

4. e2x+y d x + ey d y, where C is the triangle with vertices (0, 0), (1, 0), and (1, 1)
C
SOLUTION

(1, 1)

C
D
x
(0, 0) (1, 0)

We have P = e2x+y and Q = ey , hence


Q P
= 0 e2x+y = e2x+y
x y
y

0yx

y=x

D
x
0 1

Using Greens Theorem we get


    1 x
Q P
e2x+y d x + ey d y = dA = e2x+y d A = e2x+y d y d x
C D x y D 0 0
 1 x  1 
 e3x e2x 1 e2 e3 1
= e2x+y  dx = e3x + e2x d x = +  =
0 y=0 0 3 2 0 2 3 6


5. x 2 y d x, where C is the unit circle centered at the origin
C
SOLUTION
1198 C H A P T E R 18 F U N D A M E N TA L TH E O R E M S O F V E C T O R A N A LY S I S (ET CHAPTER 17)

D
x

In this function P = x 2 y and Q = 0. Therefore,


Q P
= 0 x 2 = x 2
x y
We obtain the following integral:
    
Q P
I = x2 y dx = dA = x 2 d A
C D x y D
We convert the integral to polar coordinates. This gives
 2  1  2  1
I = r 2 cos2 r dr d = r 3 cos2 dr d
0 0 0 0
          
2 1 sin 2 2 r 4 1 1
= cos2 d r 3 dr = +   = =
0 0 2 4 =0 4 r =0 4 4



6. F ds, where F = x + y, x 2 y and C is the boundary of the region enclosed by y = x 2 and y = x for
C
0 x 1.
SOLUTION By Greens Theorem we have
   
Q P
I = F ds = dA
C D x y
y

y= x
C

D
y = x2
x
0 1

Since P = x + y and Q = x 2 y, we have


Q P
= 2x 1
x y
Therefore,
  1  x  1 x

I = (2x 1) d A = (2x 1) d y d x = (2x 1)y  dx
D 0 x2 0 2
y=x
 1  1
= (2x 1) x x2 dx = 2x 3/2 2x 3 x 1/2 + x 2 d x
0 0

4 5/2 1 4 2 3/2 x 3 1 4 1 2 1 1
= x x x + = + =
5 2 3 3 0 5 2 3 3 30



7. F ds, where F = x 2 , x 2 and C consists of the arcs y = x 2 and y = x for 0 x 1
C
SOLUTION By Greens Theorem,
   
Q P
I = F ds = dA
C D x y
S E C T I O N 18.1 Greens Theorem (ET Section 17.1) 1199

y=x

D
y = x2
x
0 1

We have P = Q = x 2 , therefore
Q P
= 2x 0 = 2x
x y
Hence,
  1 x  1 x  1  1

I = 2x d A = 2x d y d x = 2x y  dx = 2x(x x 2 ) d x = (2x 2 2x 3 ) d x
D 0 x2 0 y=x 2 0 0

2 3 1 4 1 2 1 1
= x x  = =
3 2 0 3 2 6

8. (ln x + y) d x x 2 d y, where C is the rectangle with vertices (1, 1), (3, 1), (1, 4), and (3, 4)
C
SOLUTION By Greens Theorem,
   
Q P
I = F ds = dA
C D x y

(1, 4) (3, 4)

(1, 1) (3, 1)

We have P = ln x + y and Q = x 2 , therefore


Q P
= 2x 1
x y
Hence,
  3 4  3 4  3

I = (2x 1) d A = (2x 1) d y d x = (2x 1)y  dx = 3(2x + 1) d x
D 1 1 1 y=1 1
3

= 3(x 2 + x) = 3(12 2) = 30
1



9. The line integral of F = x 3 , 4x around the boundary of the parallelogram in Figure 15 (note the orientation)

(2, 2) (4, 2)

x
(2, 0)
FIGURE 15
1200 C H A P T E R 18 F U N D A M E N TA L TH E O R E M S O F V E C T O R A N A LY S I S (ET CHAPTER 17)

SOLUTION We have P = x 3 and Q = 4x, therefore

Q P
=40=4
x y
Hence, Greens Theorem implies
     
Q P
x 3 d x + 4x d y = dA = 4dA = 4 d A = 4 Area(D) = 4 4 = 16
C D x y D D


10. The line integral of F = e x+y , e xy around the boundary of the parallelogram in Figure 15


SOLUTION We compute the curl of F = e x+y , e xy . Here, P = e x+y and Q = e xy , hence

Q P
= e xy e x+y = e x (ey e y ).
x y
y

x=y
x=y+2

Using Greens Theorem we obtain


   2  y+2  2  y+2

F ds = e x (ey e y ) d x d y = e x (ey e y ) d x d y = e x (ey e y ) dy
C D 0 y 0 x=y
 2  2  
y+2 y y y 2 2y 2 e2y 2
= (e e )(e e ) d y = (e 1)(1 e ) d y = (e 1) y
0 0 2  y=0
  
2 e4 1 (e2 1)(5 e4 )
= (e 1) 2 =
2 2 2


11. x y d x + (x 2 + x) d y, where C is the path in Figure 16
C

(0, 1)

x
(1, 0) (1, 0)
FIGURE 16

SOLUTION

(0, 1)

C
D
x
(1, 0) (1, 0)

In the given function, P = x y and Q = x 2 + x. Therefore,


Q P
= 2x + 1 x = x + 1
x y
By Greens Theorem we obtain the following integral:
      
Q P
x y d x + (x 2 + x) d y = dA = (x + 1) d A = x dA + 1dA
C D x y D D D
S E C T I O N 18.1 Greens Theorem (ET Section 17.1) 1201

By symmetry, the positive and negative values of x cancel each other, causing the first integral to be zero. Thus,
 
21
x y d x + (x 2 + x) d y = 0 + d A = Area(D) = = 1.
C D 2

12. Let C R be the circle of radius R centered at the origin. Use Greens Theorem to determine F ds, where F is a
 C2
vector field such that F ds = 9 and curlz (F) = x 2 + y 2 for 1 x 2 + y 2 4.
C1
SOLUTION We use Greens Theorem for the annulus D between the circles C(1) and C(2) oriented as shown in the
figure.
y

C(8)

D C(3)

x
3 8

That is,
  
F ds F ds = curl(F) d x d y
C (2) C (1) D

Substituting the given information, we get


 
F ds 9 = (x 2 + y 2 ) d x d y
C (2) D
or
 
F ds = 9 + (x 2 + y 2 ) d x d y
C (2) D

We compute the double integral by converting it to polar coordinates:


  2  2  2   
r 4 2 24 14 15
F ds = 9 + r 2 r dr d = 9 + 2 r 3 dr = 9 + 2  = 9 + 2 =9+
C (2) 0 1 1 4 1 4 2

In Exercises 1316, use Eq. (5) to calculate the area of the given region.

13. The circle of radius 3 centered at the origin


SOLUTION By Eq. (5), we have

1
A= x dy y dx
2 C
We parametrize the circle by x = 3 cos , y = 3 sin , hence,

x d y y d x = 3 cos 3 cos d 3 sin (3 sin ) d = (9 cos2 + 9 sin2 ) d = 9 d

Therefore,
 
1 1 2 9
A= x dy y dx = 9 d = 2 = 9 .
2 C 2 0 2

14. The triangle with vertices (0, 0), (1, 0), and (1, 1)
SOLUTION We parametrize the segments by

O A : t, 0 , t = 0 to t = 1
AB : 1, t , t = 0 to t = 1
B O : t, t , t = 1 to t = 0
1202 C H A P T E R 18 F U N D A M E N TA L TH E O R E M S O F V E C T O R A N A LY S I S (ET CHAPTER 17)

B = (1, 1)

x
O = (0, 0) A = (1, 0)

Using Eq. (5), we obtain the following area of the triangle:


   
1 1 1 1
A= x dy y dx = y, x ds + y, x ds + y, x ds
2 C 2 OA 2 AB 2 BO
  
1 1 1 1 1 0
= 0, t 1, 0 dt + t, 1 0, 1 dt + t, t 1, 1 dt
2 0 2 0 2 1
   
1 1 1 1 1 0 1 1 1 1
= 0 dt + dt + 0 dt = dt = 1 =
2 0 2 0 2 1 2 0 2 2

15. The region between the x-axis and the cycloid parametrized by c(t) = (t sin t, 1 cos t) for 0 t 2 (Figure
17)

x
2
FIGURE 17 Cycloid.

SOLUTION By Eq. (5), the area is the following integral:



1
A= x dy y dx
2 C
where C is the closed curve determined by the segment O A and the cycloid .
y

x
O A = (2 , 0)

Therefore,
 
1 1
A= x dy y dx + x dy y dx (1)
2 OA 2 
We compute the two integrals. The segment O A is parametrized by t, 0, t = 0 to t = 2 . Hence, x = t and y = 0.
Therefore,

x d y y d x = t 0 dt 0 dt = 0

x dy y dx = 0 (2)
OA

On  we have x = t sin t and y = 1 cos t, therefore

x d y y d x = (t sin t) sin t dt (1 cos t)(1 cos t) dt


= (t sin t sin2 t 1 + 2 cos t cos2 t) dt = (t sin t + 2 cos t 2) dt

Hence,
  0  2
x dy y dx = (t sin t + 2 cos t 2) dt = (2 2 cos t t sin t) dt
 2 0
2 2
 
= 2t 2 sin t + t cos t sin t  = 2t 3 sin t + t cos t  = 6 (3)
0 0
S E C T I O N 18.1 Greens Theorem (ET Section 17.1) 1203

Substituting (2) and (3) in (1) we get


1 1
A= 0 + 6 = 3 .
2 2

16. The region between the graph of y = x 2 and the x-axis for 0 x 2
SOLUTION The boundary of the region consists of the curve  and the segments O A and AB shown in the figure.

y
B = (2, 4)
4

x
0 A = (2, 0)

By Eq. (5), the area A of the region is given by,


  
1 1 1
A= x dy y dx + x dy y dx + x dy y dx (1)
2 OA 2 AB 2 
We compute each integral separately. We use the following parametrizations:

O A : c1 (t) = (t, 0), for 0 t 2 c1 (t) = 1, 0

AB : c2 (t) = (2, t), for 0 t 4 c2 (t) = 0, 1

 : c3 (t) = (t, t 2 ) for t from 2 to 0 c3 (t) = 1, 2t

The line integrals in (1) are thus


   2
x dy y dx = y, x ds = 0, t 1, 0 dt = 0
OA OA 0
   4  4
x dy y dx = y, x ds = t, 2 0, 1 dt = 2 dt = 8
AB AB 0 0
   0  0


x dy y dx = y, x ds = t 2 , t 1, 2t dt = (t 2 + 2t 2 ) dt
  2 2
 2 
t 3 2 8
= t 2 dt = =
0 3 0 3

Substituting the integrals in (1) we obtain the following area:


 
1 1 1 8 8
A= 0+ 8+ = .
2 2 2 3 3

17. Let x 3 + y 3 = 3x y be the folium of Descartes (Figure 18).

y
2

x
2 2

FIGURE 18 Folium of Descartes.

(a) Show that the folium has a parametrization in terms of t = y/x given by

3t 3t 2
x= , y= ( < t < ) (t  = 1)
1 + t3 1 + t3
1204 C H A P T E R 18 F U N D A M E N TA L TH E O R E M S O F V E C T O R A N A LY S I S (ET CHAPTER 17)

(b) Show that

9t 2
x dy y dx = dt
(1 + t 3 )2
Hint: By the Quotient Rule,
y
x2 d = x dy y dx
x
(c) Find the area of the loop of the folium.
SOLUTION
2
(a) We show that x = 3t 3 , y = 3t 3 satisfy the equation x 3 + y 3 3x y = 0 of the folium:
1+t 1+t
 3  3
3t 3t 2 3t 3t 2
x 3 + y 3 3x y = + 3
1 + t3 1 + t3 1 + t3 1 + t3

27t 3 + 27t 6 27t 3 (1 + t 3 ) 27t 3 1 + t 3 (1 + t 3 ) 0
= 3
3
= 3
= 3
=0
(1 + t 3 ) (1 + t 3 ) (1 + t 3 ) (1 + t 3 )
2
This proves that the curve parametrized by x = 3t 3 , y = 3t 3 lies on the folium of Descartes. This parametrization
1+t 1+t
parametrizes the whole folium since the two equations can be solved for t in terms of x and y. That is,
3t
x=
1 + t3 y
t=
3t 2 x
y=
1 + t3
A glance at the graph of the folium shows that any line y = t x, with slope t, intersects the folium exactly once. Thus,
there is a one-to-one relationship between the values of t and the points on the graph.
(b) We differentiate the two sides of t = xy with respect to t. Using the Quotient Rule gives

x ddty y ddtx
1=
x2
or
 2
dy dx 3t
x y = x2 =
dt dt 1 + t3
This equality can be written in the form

9t 2
x dy y dx = 2
dt
(1 + t 3 )
(c) We use the formula for the area enclosed by a closed curve and the result of part (b) to find the required area. That is,
 
1 1 9t 2
A= x dy y dx = dt
2 C 2 0 (1 + t 3 )2

From our earlier discussion on the parametrization of the folium, we see that the loop is traced when the parameter t
is increasing along the interval 0 t < . We compute the improper integral using the substitution u = 1 + t 3 ,
du = 3t 2 dt. This gives
 R  1+R 3  3
1 9t 2 1 3 du 3 1 1+R
A= lim dt = lim = lim
2 R 0 (1 + t 3 )2 2 R 1 u2 2 R u u=1
 
3 1 3 3
= lim 1 = (1 0) =
2 R 1 + R3 2 2
y
2
C

t=
x
2 t=0 2

2
S E C T I O N 18.1 Greens Theorem (ET Section 17.1) 1205

18. Follow the procedure of Exercise 17 to find the area of the loop of the lemniscate curve with equation (x 2 + y 2 )2 =
x y (Figure 19).

0.5

0.5
x
0.5

0.5

FIGURE 19 Lemniscate.

SOLUTION We first find the parametrization of the lemniscate, determined by the parameter t = xy . We substitute
y = t x in the equation of the lemniscate and solve for x in terms of t. This gives
2 2 2
(x 2 + y 2 ) = (x 2 + t 2 x 2 ) = (1 + t 2 ) x 4
xy = x t x = t x2

We obtain the equation


2
(1 + t 2 ) x 4 = t x 2
2
(1 + t 2 ) x 2 = t

or (taking x positive)

t 1/2
x=
1 + t2
hence
t 3/2
y = tx = .
1 + t2
We obtain the parametrization

t 1/2 t 3/2
x= , y= .
1 + t2 1 + t2
One loop is traced as 0 t < .
y

t=
x
t=0

The area enclosed by one loop is given by the following line integral:

1
A= x dy y dx (1)
2 C

In Exercise 17 we showed the relation t = xy implies that

x d y y d x = x 2 dt
1/2
Now x = t 2 , hence
1+t
t
x dy y dx = 2
dt
(1 + t 2 )

We substitute in (1) and compute the resulting improper integral using the substitution u = 1 + t 2 , du = 2t dt. We get
   R  1+R 2 1
1 1 t 1 t dt 1 2 du du
A= x dy y dx = 2
dt = lim 2
= lim
2 C 2 0 (1 + t 2 ) 2 R 0 (1 + t 2 ) 2 R 1 u2
1206 C H A P T E R 18 F U N D A M E N TA L TH E O R E M S O F V E C T O R A N A LY S I S (ET CHAPTER 17)

 2  
1 1 1+R 1 1 1 1
= lim  = lim 1 = (1 0) =
4 R u u=1 4 R 1 + R2 4 4

19. Show that if C is a simple closed curve, then


 
y d x = x dy
C C
and both integrals are equal to the area enclosed by C.

SOLUTION We show that C y d x + x d y = 0 by showing that the vector field F = y, x is conservative. Indeed, since
P = y and Q = x, we have Q P
x = 1 and y = 1. Therefore, the cross partials are equal and therefore F is conservative.
By the formula for the area enclosed by a simple closed curve, the area enclosed by C is
  
1 1 1
A= x dy y dx = x dy + y d x
2 C 2 C 2 C
Using the equality obtained above, we have
   
1 1
A= x dy + x dy = x dy = y d x.
2 C 2 C C C

20. For the vector fields (A)(D) in Figure 20, state whether the curl at the origin appears to be positive, negative, or
zero.
y y

x x

(A) (B)

y y

x x

(C) (D)
FIGURE 20

SOLUTION The vector field (A) does not have spirals nor it is a shear flow. Therefore, the curl appears to be zero. The
vector field (B) rotates in the clockwise direction, hence we expect the curl to be negative. The vector field (C) rotates
counterclockwise around the origin, indicating a positive curl. Finally, in the vector field (D) the fluid flows straight
toward the origin without spiraling. We expect the curl to be zero.



21. Let F = 2xe y , x + x 2 e y and let C be the quarter-circle path from A to B in Figure 21. Evaluate I = F ds as
C
follows:
(a) Find a function (x, y) such that F = G + , where G = 0, x.
(b) Show that the line integrals of G along the segments O A and O B are zero.
(c) Use Greens Theorem to show that

I = (B) ( A) + 4

and evaluate I .
y

B = (0, 4)

x
O A = (4, 0)

FIGURE 21
S E C T I O N 18.1 Greens Theorem (ET Section 17.1) 1207

SOLUTION
(a) We need to find a potential function (x, y) for the difference
   
F G = 2xe y , x + x 2 e y 0, x = 2xe y , x 2 e y

We let (x, y) = x 2 e y .
(b) We use the parametrizations AO : t, 0, 0 t 4 and O B : 0, t, 0 t 4 to evaluate the integrals of
G = 0, x. We get
  4  4
G ds = 0, t 1, 0 dt = 0 dt = 0
OA 0 0
  4  4
G ds = 0, 0 0, 1 dt = 0 dt = 0
OB 0 0

B = (0, 4)

x
O A = (4, 0)

(c) Since F G = , we have


   
(F G) ds = (B) ( A) = F ds G ds = I G ds
C C C C
That is,

I = (B) ( A) + G ds (1)
C
To compute the line integral on the right-hand side, we rewrite it as
   
G ds = G ds G ds G ds
C B O+O A+C BO OA

Using part (b) we may write


 
G ds = G ds (2)
C B O+O A+C

We now use Greens Theorem. Since G = 0, x, we have P = 0 and Q = x, hence Q P
x y = 1 0 = 1. Thus,
 
42
G ds = 1 d A = Area(D) = = 4 (3)
B O+O A+C D 4
Combining (1), (2), and (3), we obtain

I = (B) ( A) + 4

Since (x, y) = x 2 e y , we conclude that

I = (0, 4) (4, 0) + 4 = 0 42 e0 + 4 = 4 16.


y

B = (0, 4)
C

x
O A = (4, 0)



22. Compute the line integral of F = x 3 , 4x along the path from A to B in Figure 22. Hint: To save work, use Greens
Theorem to relate this line integral to the line integral along the vertical path from B to A.
1208 C H A P T E R 18 F U N D A M E N TA L TH E O R E M S O F V E C T O R A N A LY S I S (ET CHAPTER 17)

A = (1, 0)
x

B = (1, 1)

FIGURE 22

SOLUTION We denote by C the path from A to B, and D is the region enclosed by C and the segment B A.
y
C 1

D
A = (1, 0)
x
2

B = (1, 1)

By Greens Theorem, we have


     
Q P
F ds = F ds + F ds = dA
C +B A C BA D x y
or
    
Q P
F ds = dA + F ds (1)
C D x y AB

We compute the integrals on the right-hand side. We parametrize the segment AB by 1, t, with t from 0 to 1. We
get
  1  1  0
F ds = 1, 4 0, 1 dt = 4 dt = 4 dt = 4 (2)
AB 0 0 1

Since Q = 4x and P = x 3 , we have Q P


x y = 4 0 = 4. Hence,
    
Q P
dA = 4dA = 4 1 d A = 4Area(D) = 4(1 3 + 1 1) = 16 (3)
D x y D D
Substituting (2) and (3) in (1), we get

F ds = 4 + 16 = 20
C

23. Evaluate I = (sin x + y) d x + (3x + y) d y for the nonclosed path ABCD in Figure 23. Hint: Use the method of
C
Exercise 22.
y

D = (0, 6)

(2, 4)

(2, 2)
A = (0, 0)
x

FIGURE 23

SOLUTION

D = (0, 6)
C
C = (2, 4)
D
B = (2, 2)
A = (0, 0)
x
S E C T I O N 18.1 Greens Theorem (ET Section 17.1) 1209

Let F = sin x + y, 3x + y, hence P = sin x + y and Q = 3x + y. We denote by C1 the closed path determined by C
and the segment D A. Then by Greens Theorem,
     
Q P
P dx + Q dy = dA = (3 1) d A = 2 d A = 2 Area(D) (1)
C1 D x y D D

The area of D is the area of the trapezoid ABC D, that is,


 
BC + AD h (2 + 6) 2
Area(D) = = = 8.
2 2
y

D = (0, 6)

C = (2, 4)
h

B = (2, 2)
A = (0, 0)
x

Combining with (1) we get



P d x + Q d y = 2 8 = 16
C1

Using properties of line integrals, we have


 
P dx + Q dy + P d x + Q d y = 16 (2)
C DA

We compute the line integral over D A, using the parametrization

D A : x = 0, y = t, t varies from 6 to 0.

We get
  0  0
d
P dx + Q dy = F(0, t) 0, t dt = sin 0 + t, 3 0 + t 0, 1 dt
DA 6 dt 6
 0  0 
t 2 0
= t, t 0, 1 dt = t dt =  = 18
6 6 2 t=6

We substitute in (2) and solve for the required integral:


 
P d x + Q d y 18 = 16 or P d x + Q d y = 34.
C C

24. Estimate the circulation of a vector field F around a circle of radius R = 0.1, assuming that curlz (F) takes the value
4 at the center of the circle.
SOLUTION We estimate the circulation by

F ds curl(F)(P)Area(D) (1)
C
C

1
P

We are given that curl(F)(P) = 4. The area of the disk of radius R = 0.1 is 0.12 = /100. Substituting in (1) gives
the estimation


F ds 4 =
C 100 25
1210 C H A P T E R 18 F U N D A M E N TA L TH E O R E M S O F V E C T O R A N A LY S I S (ET CHAPTER 17)

25. Let F be the velocity field. Estimate the circulation of F around a circle of radius R = 0.05 with center P, assuming
that curlz (F)(P) = 3. In which direction would a small paddle placed at P rotate? How fast would it rotate (in radians
per second) if F is expressed in meters per second?
SOLUTION We use the following estimation:

F ds curl(F)(P)Area(D) (1)
C
C

D
0.05
P

We are given that curl(F)(P) = 3. Also, the area of the disk of radius R = 0.05 is 0.052 = 0.0025 . Therefore, we
obtain the following estimation:

F ds 3 0.0025 0.024.
C
Since the curl is negative, the paddle would rotate in the clockwise direction. Using the formula |curl(F)| = 2 , we see
that the angular speed is = 1.5 radians per second.
 
26. Referring to Figure 24, suppose that F ds = 12. Use Greens Theorem to determine F ds, assuming that
C2 C1
curlz (F) = 3 in D.

C1 3

C2 D
x
2 5

FIGURE 24

SOLUTION By Greens Theorem,


  
F ds F ds = curl(F) d x d y
C1 C2 D

Substituting the given information gives


  
F ds 12 = 3 d x d y = 3 1 d x d y = 3 Area(D)
C1 D D

Hence,

F ds 12 = 3 Area(D) (1)
C1

We compute the area of D as the difference between the area of the rectangle and the area of the inner disk. That is,

Area(D) = 6 10 22 = 60 4

Substituting in (1) we get



F ds = 12 3(60 4 ) = 12 168 130.3.
C1

27. Referring to Figure 25, suppose that


 
F ds = 3 , F ds = 4
C2 C3

Use Greens Theorem to determine the circulation of F around C1 , assuming that curlz (F) = 9 on the shaded region.
S E C T I O N 18.1 Greens Theorem (ET Section 17.1) 1211

C1

D
C3 C2
1 1

FIGURE 25

SOLUTION We must calculate C1 F ds. We use Greens Theorem for the region D between the three circles C1 , C2 ,

and C3 . Because of orientation, the line integrals C2 F ds = C2 F ds and C3 F ds = C3 F ds must be used
in applying Greens Theorem. That is,
   
F ds F ds F ds = curl(F) d A
C1 C2 C3 D

We substitute the given information to obtain


  
F ds 3 4 = 9dA = 9 1 d A = 9 Area(D) (1)
C1 D D

The area of D is computed as the difference of areas of discs. That is,

Area(D) = 52 12 12 = 23

We substitute in (1) and compute the desired circulation:



F ds 7 = 9 23
C1
or

F ds = 214 .
C1

28. Area of a Polygon Greens Theorem leads to a convenient formula for the area of a polygon.
(a) Let C be the line segment joining (x 1 , y1 ) to (x 2 , y2 ). Show that

1 1
y d x + x d y = (x 1 y2 x 2 y1 )
2 C 2
(b) Prove that the area of the polygon with vertices (x 1 , y1 ), (x 2 , y2 ), . . . , (x n , yn ) is equal [where we set
(x n+1 , yn+1 ) = (x 1 , y1 )] to

1 n
(xi yi+1 xi+1 yi )
2 i=1

SOLUTION
(a) We parametrize the segment from (x 1 , y1 ) to (x 2 , y2 ) by

x = t x 2 + (1 t)x 1 , y = t y2 + (1 t)y1 , 0t 1

Then, d x = (x 2 x 1 ) dt and d y = (y2 y1 ) dt. Therefore,

y d x + x d y = (t y2 (1 t)y1 ) (x 2 x 1 ) dt + (t x 2 + (1 t)x 1 ) (y2 y1 ) dt


= (t y2 x 2 + t y2 x 1 y1 x 2 (1 t) + (1 t)y1 x 1 + t x 2 y2 t x 2 y1 + (1 t)x 1 y2 (1 t)x 1 y1 ) dt
= (x 1 y2 x 2 y1 ) dt

We obtain the following integral:


  1
1 1 1 1  1
y d x + x d y = (x 1 y2 x 2 y1 ) dt = (x 1 y2 x 2 y1 )t  = (x 1 y2 x 2 y1 )
2 C 2 0 2 t=0 2

An = (xn , yn)
An 1 = (xn 1, yn 1)
A1 = (x1, y1)

A2 = (x2, y2)
1212 C H A P T E R 18 F U N D A M E N TA L TH E O R E M S O F V E C T O R A N A LY S I S (ET CHAPTER 17)

(b) Let Ai = (xi , yi ), i = 1, 2, . . . , n, and let C be the closed curve determined by the polygon. By the formula for the
area enclosed by a simple closed curve, the area of the polygon is

1
A= y d x + x d y
2 C
We use additivity of line integrals and the result in part (a) to write the integral as follows:
 

1 n1

A= y d x + x d y + y d x + x d y
2 i=1 Ai Ai+1 An A1
 

1 n1
= (xi yi+1 xi+1 yi ) + (x n y1 x 1 yn )
2 i=1


1 n1 1
= (xi yi+1 xi+1 yi ) + (x n y1 x 1 yn )
2 i=1 2

If we define (x n+1 , yn+1 ) = (x 1 , y1 ), we obtain the sum

1 n
A= (xi yi+1 xi+1 yi ).
2 i=1

29. Use the result of Exercise 28 to compute the areas of the polygons in Figure 26. Check your result for the area of the
triangle in (A) using geometry.

y y
(3, 5)
5 5
4 4
(2, 3) (1, 3) (5, 3)
3 3
2 2
(3, 2)
1 (1, 1) 1
(2, 1) (5, 1)
x x
1 2 3 4 5 3 2 1 1 2 3 4 5
(A) (B)
FIGURE 26

SOLUTION
(a) The vertices of the triangle are

(x 1 , y1 ) = (x 4 , y4 ) = (2, 1), (x 2 , y2 ) = (5, 1), (x 3 , y3 ) = (2, 3)


y

5
4
(2, 3)
3
2
1
(2, 1) (5, 1)
x
1 2 3 4 5

Using the formula obtained in Exercise 28, the area of the triangle is the following sum:
1 
A= (x y x 2 y1 ) + (x 2 y3 x 3 y2 ) + (x 3 y1 x 1 y3 )
2 1 2
1  1
= (2 1 5 1) + (5 3 2 1) + (2 1 2 3) = (3 + 13 4) = 3
2 2
We verify our result using the formula for the area of a triangle:
1 1
A= bh = (5 2) (3 1) = 3
2 2
(b) The vertices of the polygon are

(x 1 , y1 ) = (x 6 , y6 ) = (1, 1)
(x 2 , y2 ) = (1, 3)
(x 3 , y3 ) = (3, 2)
S E C T I O N 18.1 Greens Theorem (ET Section 17.1) 1213

(x 4 , y4 ) = (5, 3)
(x 5 , y5 ) = (3, 5)
y
(3, 5)
5
4
(1, 3) (5, 3)
3
2
(3, 2)
(1, 1) 1
x
3 2 1 1 2 3 4 5

Using the formula in part (a), the area of the polygon is the following sum:
1 
A= (x 1 y2 x 2 y1 ) + (x 2 y3 x 3 y2 ) + (x 3 y4 x 4 y) + (x 4 y5 x 5 y4 ) + (x 5 y1 x 1 y5 )
2
1    
= (1 3 1 1) + (1 2 3 3) + (3 3 5 2) + 5 5 (3) 3 + 3 1 (1) 5
2
1
= (4 7 1 + 34 + 2) = 12
2

Further Insights and Challenges


In Exercises 3031, let F be the vortex vector field [defined for (x, y)  = (0, 0)]:
 
y x
F= 2 ,
x + y2 x 2 + y2
30. (a) Show that curlz (F)(P) = 0 for all P  = (0, 0).
(b) Show that the circulation of F around the circle C R of radius R centered at the origin is equal to 2 for all R.
SOLUTION
y
(a) We find Q P x
x and y for Q = x 2 +y 2 and P = x 2 +y 2 . Using the Quotient Rule we get
 
Q x 1 (x 2 + y 2 ) x 2x y2 x 2
= = =
x x x + y 2 2
(x 2 + y 2 )
2
(x 2 + y 2 )
2

 
P y 1 (x 2 + y 2 ) + y 2y y2 x 2
= = =
y y x + y 2 2
(x 2 + y 2 )
2
(x 2 + y 2 )
2

Since Q P
x = y , the curl of F is identically zero for (x, y)  = (0, 0):

Q P
curl(F) = =0
x y
(b) Let C R be the circle of radius R centered at the origin. We parametrize C R by

C( ) = R cos , R sin  , 0 < 2

Hence,
   
R sin R cos sin cos
F (C( )) = , = ,
R 2 cos2 + R 2 sin2 R 2 cos2 + R 2 sin2 R R
 
sin cos
F (C( )) C  ( ) = , R sin , R cos  = sin2 + cos2 = 1
R R
The line integral is thus
  2  2  2

F ds = F (C( )) C  ( ) d = 1 d =  = 2 .
C 0 0 0

31. Prove that if C is a simple closed curve whose interior contains the origin, then F ds = 2 (Figure 27). Hint:
C
Apply Greens Theorem to the domain between C and C R where R is so small that C R is contained in C.
1214 C H A P T E R 18 F U N D A M E N TA L TH E O R E M S O F V E C T O R A N A LY S I S (ET CHAPTER 17)

C
CR
x

FIGURE 27

SOLUTION Let R > 0 be sufficiently small so that the circle C R is contained in C.

C
CR D
x

Let D denote the region between C R and C. We apply Greens Theorem to the region D. The curve C is oriented
counterclockwise and C R is oriented clockwise. We have
  
F ds + F ds = curl(F) d A (1)
C CR D

From Exercise 30(b) we know that C R F ds = 2 . Since D does not contain the origin, we have by part (a) of
Exercise 30, curl (F) = 0 on D. Substituting in (1) we obtain
 
F ds 2 = 0dA = 0
C D
or

F ds = 2 .
C

In Exercises 3234, the conjugate of a vector field F = P, Q is the vector field F = Q, P.

32. Explain the following statement: F is the vector field obtained by rotating the vectors of F counterclockwise through
an angle of 2 .
SOLUTION We show that the vector b = y, x is obtained by rotating the vector a = x, y counterclockwise. First,

notice that a and b are on the same circle centered at the origin, since a = b = x 2 + y 2 . Notice also that since
a b = yx + x y = 0, the angle between the two vectors is 2 , and the two vectors are in adjacent quadrants. In addition,
the x-coordinate of b and the y-coordinate of a have opposite signs. By these reasonings we conclude that b is obtained
by rotating a counterclockwise by 2 .
33. The normal component of F at a point P on a simple closed C is the quantity F(P) n(P), where n(P) is the
 C is defined as the line integral of the normal component
outward-pointing unit normal vector. The flux of F across curve
around C (Figure 28). Show that the flux across C is equal to F ds.
C

P
n(P)

FIGURE 28 The flux of F is the integral of the normal component F n around the curve.

SOLUTION We must show that if C is a simple closed curve, then


 
(F n)ds = F ds
C C
S E C T I O N 18.1 Greens Theorem (ET Section 17.1) 1215

By the definition of the vector line integral, the line integral of the vector field F over C is
 
F ds = (F T) ds (1)
C C

In Exercise 32 we showed that F is a rotation of F by 2 counterclockwise. Therefore, the angle between F and the
 
tangent T is equal to the angle between F and the normal n. Also, F  = F. Hence,
 
F T = F  T cos = F cos
F n = Fn cos = F cos

Since the dot products are equal, we conclude that


 

(F T) ds = (F n)ds (2)
C C
Combining (1) and (2) we obtain
 
F ds = (F n)ds.
C C
T
F*
n
q
q
F

P Q
34. Define div(F) = + . Use Greens Theorem to prove that for any simple closed curve C,
x y

Flux across C = div(F) d A
D
where D is the region enclosed by C. This is a two-dimensional version of the Divergence Theorem discussed in Section
18.3.
SOLUTION Since F = P, Q and F = Q, P, we have

P Q
div(F) = +
x y
P P Q
curl(F ) = (Q) = +
x y x y
Therefore,

div(F) = curl(F ) (1)

Using Exercise 33, the flux of F across C is



flux of F across C = F ds (2)
C
Greens Theorem and (1) imply that
  
F ds = curl(F ) d A = div(F) d A (3)
C D D
Combining (2) and (3) we have

flux of F across C = div(F) d A.
D

In Exercises 3538, the Laplace operator  is defined by

2 2
 = + 2 9
x 2 y

35. Let F = . Show that curlz (F ) =  , where F is the conjugate vector field (defined in Exercises 3234).
1216 C H A P T E R 18 F U N D A M E N TA L TH E O R E M S O F V E C T O R A N A LY S I S (ET CHAPTER 17)

SOLUTION For a vector field F = P, Q, the conjugate vector field is F = Q, P. By the given information,
   

F = = , F = ,
x y y x

We compute the curl of F :


   
2 2
curl(F ) = = + = 
x x y y x2 y 2

36. Let n denote the outward-pointing unit normal vector to a simple closed C. The normal derivative of a function ,

denoted , is the directional derivative Dn ( ) = n. Prove the formula
n
 

ds =  d A
C n D

where D is the domain enclosed by C. Hint: Let F = . Show that = F T, where T is the unit tangent vector
n
pointing in the counterclockwise direction along C, and apply Greens Theorem.
SOLUTION In Exercise 32 we showed that for any vector field F, F is a rotation of F by 2 counterclockwise. The unit
tangent en is a rotation of n by 2 counterclockwise.

F*
n
en
q q
F

 
These properties imply that the angle between F and n is equal to the angle between F and en , and F = F .
Therefore,

F n = Fn cos = F cos


  F n = F en
F en = F  en  cos = F cos

Now, if F = , then

= n = F n = F en
n

By the definition of the vector line integral C F ds = C (F en ) ds. Therefore,
  

ds = (F en ) ds = F ds
C n C C

Using Greens Theorem and the equality curl(F ) =  obtained in Exercise 35, we get
   

ds = F ds = curl(F ) d A =  d A.
C n C D D

37. Let P = (a, b) and let C(r ) be the circle of radius r centered at P. The average value of a continuous function on
C(r ) is defined as the integral
 2
1
I (r ) = (a + r cos , b + r sin ) d
2 0
(a) Show that

(a + r cos , b + r sin ) = (a + r cos , b + r sin )
n r
(b) Use differentiation under the integral sign to prove that

d 1
I (r ) = ds
dr 2 r C (r ) n
S E C T I O N 18.1 Greens Theorem (ET Section 17.1) 1217

(c) Use Exercise 36 to conclude that



d 1
I (r ) =  d A
dr 2 r D (r )

where D(r ) is the interior of C(r ).


SOLUTION In this solution, r (a + r cos , b + r sin ) denotes the partial derivative r computed at (a + r cos , b +
(a + r cos , b + r sin ) is the derivative of the composite function.
r sin ), whereas r

(a) Since n = n, we first express the gradient vector in terms of polar coordinates. We use the Chain Rule and
the derivatives:
sin cos
x = , y = , r x = cos , r y = sin
r r
We get
 
sin
x = r r x + x = r cos +
r
 
cos
y = r r y + y = r sin + (1)
r
Hence,
 
sin cos
= r cos , r sin +
r r
We use the following parametrization for C(r ):

C(r ) : c( ) = a + r cos , b + r sin  , 0 2

The unit normal vector is

n = cos , sin  .

We compute the dot product:


 
sin cos
= n = r cos , r sin + cos , sin 
n r r
sin cos cos sin  
= r cos2 + r sin2 + = r cos2 + sin2 = r
r r
That is,

(a + r cos , b + r sin ) = r (a + r cos , b + r sin ) (2)
n
(b) We compute the following derivative using the Chain Rule and (1):

(a + r cos , b + r sin ) = x (a + r cos ) + y (b + r sin )
r r r
   
sin cos
= r cos cos + r sin + sin
r r
sin cos cos sin
= r cos2 + r sin2 +
r r
= r (a + r cos , b + r sin ) (3)

We now differentiate I (r ) under the integral sign, and use (3) and (2) to obtain
 2  2
d 1 1
I (r ) = (a + r cos , b + r sin ) d = r (a + r cos , b + r sin ) d
dr 2 0 r 2 0
 2
1
= (a + r cos , b + r sin ) d (4)
2 0 n

On the other hand, since c ( ) = r sin , r cos , we have


  2  2
 
ds = (a + r cos , b + r sin ) c ( ) d = (a + r cos , b + r sin )r d
C (r ) n 0 n 0 n
1218 C H A P T E R 18 F U N D A M E N TA L TH E O R E M S O F V E C T O R A N A LY S I S (ET CHAPTER 17)

 2

=r (a + r cos , b + r sin ) d (5)
0 n
Combining (4) and (5) we get

d 1
I (r ) = ds.
dr 2 r C (r ) n

(c) We combine the result of part (b) and Exercise 36 to conclude


 
d 1 1
I (r ) = ds =  d A.
dr 2 r C (r ) n 2 r D (r )

38. Prove that m(r ) I (r ) M(r ), where m(r ) and M(r ) are the minimum and maximum values of on C(r ). Then
use the continuity of to prove that lim I (r ) = (P).
r 0
SOLUTION I (r ) is defined by
 2
1
I (r ) = (a + r cos , b + r sin ) d
2 0
The points on C(r ) have the form (a + r cos , b + r sin ). Therefore, since m(r ) and M(r ) are the minimum and
maximum values of on C(r ), we have for all 0 2 ,

m(r ) (a + r cos , b + r sin ) M(r )

Using properties of integrals (Eq. (6) in Section 5.2), we conclude that


 2
2 m(r ) (a + r cos + b + r sin ) 2 M(r )
0
Dividing by 2 we obtain

m(r ) I (r ) M(r ) (1)

Now, since is continuous and the functions sin and cos are bounded for all 0 2 , the following holds:
 
lim (a + r cos , b + r sin ) = lim (a + r cos , b + r sin ) = (a, b)
r 0 r 0

which means that for  > 0 there exists > 0 so that

| (a + r cos , b + r sin ) (a, b)| < 

for all 0 2 , whenever 0 < r < . Hence also

lim m(r ) = lim M(r ) = (a, b) (2)


r 0 r 0

Combining (1), (2), and the Squeeze Theorem, we obtain the following conclusion:

lim I (r ) = (a, b).


r 0

In Exercises 3940, let D be the region bounded by a simple closed curve C. A function (x, y) on D (whose second-
order partial derivatives exist and are continuous) is called harmonic if  = 0, where  is the Laplace operator
defined in Eq. (9).

39. Use the results of Exercises 37 and 38 to prove the mean-value property of harmonic functions: If is harmonic,
then I (r ) = (P) for all r .
SOLUTION In Exercise 37 we showed that

d 1
I (r ) =  d A
dr 2 r D
If is harmonic,  = 0. Therefore the right-hand side of the equality is zero, and we get
d
I (r ) = 0
dr
We conclude that I (r ) is constant, that is, I (r ) has the same value for all r . The constant value is determined by the
limit limr 0 I (r ) = (P) obtained in Exercise 38. That is, I (r ) = (P) for all r .
S E C T I O N 18.2 Stokes Theorem (ET Section 17.2) 1219

40. Show that f (x, y) = x 2 y 2 is harmonic. Verify the mean-value property for f (x, y) directly [expand f (a +
r cos , b + r sin ) as a function of and compute I (r )]. Show that x 2 + y 2 is not harmonic and does not satisfy the
mean-value property.
We show that the function f (x, y) = x 2 y 2 is harmonic by showing that  f = 2f + 2f = 0. We
2 2
SOLUTION
x y
have
f f
= 2x, = 2y
x y
2 f 2 f
= 2, = 2
x2 y 2
Hence,
2 f 2 f
f = + =22=0
x2 y 2
We now verify the mean-value property for f . That is, we show that for all r ,
 2
1
I f (r ) = f (a + r cos , b + r sin ) d = f (a, b)
2 0
We compute the integrand:
f (a + r cos , b + r sin ) = x 2 y 2 = (a + r cos )2 (b + r sin )2

= a 2 + 2ar cos + r 2 cos2 b2 + 2br sin + r 2 sin2

= a 2 b2 + 2r (a cos b sin ) + r 2 cos 2


We compute the integral:
 2
2 I f (r ) = a 2 b2 + 2r (a cos b sin ) + r 2 cos 2 d
0
 2
r2 
= (a 2 b2 ) + 2ar sin + 2br cos + sin 2  = 2 (a 2 b2 )
2 =0
Hence,
I f (r ) = a 2 b2

However, we have f (a, b) = a 2 b2 . Hence, for all r , I f (r ) = f (a, b), which proves the mean-value property for f .
For g(x, y) = x 2 + y 2 we have
gx x = 2, g yy = 2, and g = 2 + 2 = 4  = 0.
We check the mean value property:
 2  2
1 1
I g (r ) = g(a + r cos , b + r sin ) d = (a + r cos )2 + (b + r sin )2 d
2 0 2 0
 2
1
= a 2 + b2 + 2r (a cos + b sin ) + r 2 d = a 2 + b2 + r 2  = a 2 + b2 = (a, b)
2 0
The mean value property does not hold for g.

18.2 Stokes Theorem (ET Section 17.2)


Preliminary Questions
1. Indicate with an arrow the boundary orientation of the boundary curves of the surfaces in Figure 13, oriented by the
outward-pointing normal vectors.

n n

(A) (B)
FIGURE 13
1220 C H A P T E R 18 F U N D A M E N TA L TH E O R E M S O F V E C T O R A N A LY S I S (ET CHAPTER 17)

SOLUTION The indicated orientation is defined so that if the normal vector is moving along the boundary curve, the
surface lies to the left. Since the surfaces are oriented by the outward-pointing normal vectors, the induced orientation is
as shown in the figure:

n n

(A) (B)

2. Let F = curl(A). Which of the following are related by Stokes Theorem?


(a) The circulation of A and flux of F.
(b) The circulation of F and flux of A.
SOLUTION Stokes Theorem states that the circulation of A is equal to the flux of F. The correct answer is (b).
3. What is the definition of a vector potential?
SOLUTION A vector field A such that F = curl(A) is a vector potential for F.
4. Which of the following statements is correct?
(a) The flux of curl(F) through every oriented surface is zero.
(b) The flux of curl(F) through every closed, oriented surface is zero.
SOLUTION Statement (b) is the correct statement. The flux of curl(F) through an oriented surface is not necessarily
zero, unless the surface is closed.
5. Which condition on F guarantees that the flux through S1 is equal to the flux through S2 for any two oriented
surfaces S1 and S2 with the same oriented boundary?
SOLUTION If F has a vector potential A, then by a corollary of Stokes Theorem,
 
F ds = A ds
S C
Therefore, if two oriented surfaces S1 and S2 have the same oriented boundary curve, C, then
   
F ds = A ds and F ds = A ds
S1 C S2 C

Hence,
 
F ds = F ds
S1 S2

Exercises
In Exercises 14, calculate curl(F).


1. F = z y 2 , x + z 3 , y + x 2
SOLUTION We have
 
 i j k 
 
 
 

curl(F) =   = (1 3z 2 )i (2x 1)j + (1 + 2y)k = 1 3z 2 , 1 2x, 1 + 2y

 x y z 
 
 z y2 x + z3 y+x 
2

 
y y z
2. F = , ,
x z x
SOLUTION The curl is the following vector:
 
 i j k 

         
 
curl(F) =   = 0 y i z 0 j + 0 1 k = y , z , 1

 x y z  z2 x2 x z2 x 2 x
 y 
 y z 
x z x
S E C T I O N 18.2 Stokes Theorem (ET Section 17.2) 1221



3. F = e y , sin x, cos x
SOLUTION We have
 
 i j k 
 
 
 

curl(F) =   = 0i ( sin x)j + (cos x e y )k = 0, sin x, cos x e y
 x y z 
 
 ey sin x cos x 

 
x y
4. F = , , 0
x 2 + y2 x 2 + y2
SOLUTION

 
 i j k 
 
   
  2x y
  2x y
curl(F) =  x y z  = 0i 0j + + k=0
 (x 2 + y 2 )
2
(x 2 + y 2 )
2
 x y 
 0 

x 2 + y2 x 2 + y2

In Exercises 58, verify Stokes Theorem for the given vector field and surface, oriented with an upward-pointing normal.

5. F = 2x y, x, y + z, the surface z = 1 x 2 y 2 for x 2 + y 2 1


SOLUTION We must show that
 
F ds = curl (F) dS
C S

Step 1. Compute the line integral around the boundary curve. The boundary curve C is the unit circle oriented in the
counterclockwise direction. We parametrize C by

(t) = (cos t, sin t, 0), 0 t 2

Then,
 
F (t) = 2 cos t sin t, cos t, sin t
 (t) =  sin t, cos t, 0
  
F (t) (t) = 2 cos t sin t, cos t, sin t  sin t, cos t, 0 = 2 cos t sin2 t + cos2 t

We obtain the following integral:


  2 
 2 2  2 sin3 t t sin 2t 2
F ds = 2 cos t sin t + cos t dt = + + = (1)
C 0 3 2 4 0

Step 2. Compute the flux of the curl through the surface. We parametrize the surface by
 
( , t) = t cos , t sin , 1 t 2 , 0 t 1, 0 2

We compute the normal vector:



T = = t sin , t cos , 0


Tt = = cos , sin , 2t
t
 
 i j k 

T Tt =  t sin t cos 0  = (2t 2 cos )i (2t 2 sin )j t (sin2 + cos2 )k
 cos sin 2t 
1222 C H A P T E R 18 F U N D A M E N TA L TH E O R E M S O F V E C T O R A N A LY S I S (ET CHAPTER 17)

= (2t 2 cos )i (2t 2 sin )j tk

Since the normal is always supposed to be pointing upward, the z-coordinate of the normal vector must be positive.
Therefore, the normal vector is


n = 2t 2 cos , 2t 2 sin , t

We compute the curl:


 
 i j k 
 
 
 
curl(F) =   = i + (1 2x)k = 1, 0, 1 2x
 x y z 
 
 2x y x y+z 

We compute the curl in terms of the parameters:

curl(F) = 1, 0, 1 2t cos 

Hence,
 
curl(F) n = 1, 0, 1 2t cos  2t 2 cos , 2t 2 sin , t = 2t 2 cos + t 2t 2 cos = t

The surface integral is thus


  2  1  1 
t 2 1
curl(F) dS = t dt d = 2 t dt = 2 = (2)
S 0 0 0 2 0
The values of the integrals in (1) and (2) are equal, as stated in Stokes Theorem.
x y
6. F = yz, 0, x, the portion of the plane + + z = 1 where x, y, z 0
2 3
SOLUTION
Step 1. Compute the integral around the boundary curve. The boundary curve C consists of the segments C1 , C2 , and C3
shown in the figure:
z

C3 C2

y
2 C1 3
x

We parametrize the segments by

C1 : 1 (t) = (2 2t, 3t, 0), t from 0 to 1


C2 : 2 (t) = (0, 3 3t, t), t from 0 to 1
C3 : 3 (t) = (2t, 0, 1 t), t from 0 to 1

We compute the following values:


 
F 1 (t) = yz, 0, x = 0, 0, 2 2t , 1 (t) = 2, 3, 0
   
F 2 (t) = yz, 0, x = 3t 3t 2 , 0, 0 , 2 (t) = 0, 3, 1
 
F 3 (t) = yz, 0, x = 0, 0, 2t , 3 (t) = 2, 0, 1

Hence,
 
F 1 (t) 1 (t) = 0, 0, 2 2t 2, 3, 0 = 0
   
F 2 (t) 2 (t) = 3t 3t 2 , 0, 0 0, 3, 1 = 0
 
F 3 (t) 3 (t) = 0, 0, 2t 2, 0, 1 = 2t

We obtain the following integral:


     1 1

F ds = F ds + F ds + F ds = 0 + 0 + 2t dt = t 2  = 1 (1)
C C1 C2 C3 0 0
S E C T I O N 18.2 Stokes Theorem (ET Section 17.2) 1223

Step 2. Compute the curl.


 
 i j k 
 
 
 
curl(F) =   = (1 y)j + (0 z)k = 0, y 1, z
 x y z 
 
 yz 0 x 

Step 3. Compute the flux of the curl through the surface. We parametrize the portion of the plane by
x y
(x, y) = x, y, 1 for (x, y) D
2 3
y
3

3x
y= +3
2

x
0 2

We find the normal vector:


 
 1
Tx = = 1, 0,
x 2
 
 1
Ty = = 0, 1,
y 3
 
 i j k 
  
  1 1 1 1
Tx T y =  1 0 12  = i + j + k = , , 1
  2 3 2 3
 0 1 1 
3

The upward pointing normal is


 
1 1
n= , ,1 .
2 3
We compute the curl(F) in terms of the parameters x and y:
 x y   x y 
curl(F) = 0, y 1, 1 = 0, y 1, + 1
2 3 2 3
We obtain the following integral:
   x y  1 1   
x 2y 4

curl(F) dS = + 1 , ,1 dA =
0, y 1, + dA
S D 2 3 2 3 D 2 3 3
 2  3x/2+3    2 
x 2y 4 xy y2 4y 3x/2+3
= + dy dx = + dx
0 0 2 3 3 0 2 3 3  y=0
 2 2
x x2 
= 1 dx = x  = 1 (2)
0 2 4 0

The integrals in (1) and (2) are equal as stated in Stokes Theorem.


7. F = e yz , 0, 0 , the square with vertices (1, 0, 1), (1, 1, 1), (0, 1, 1), and (0, 0, 1)
SOLUTION
Step 1. Compute the integral around the boundary curve. The boundary consists of four segments C1 , C2 , C3 , and C4
shown in the figure:
1224 C H A P T E R 18 F U N D A M E N TA L TH E O R E M S O F V E C T O R A N A LY S I S (ET CHAPTER 17)

(0, 0, 1)
C4
C1 (0, 1, 1)
S
(1, 0, 1)
C3
C2
(1, 1, 1)

We parametrize the segments by

C1 : 1 (t) = (t, 0, 1), 0t 1


C2 : 2 (t) = (1, t, 1), 0t 1
C3 : 3 (t) = (1 t, 1, 1), 0t 1
C4 : 4 (t) = (0, 1 t, 1), 0t 1

We compute the following values:


 
 
F 1 (t) = e yz , 0, 0 = e1 , 0, 0
 
 
F 2 (t) = e yz , 0, 0 = et1 , 0, 0
 

F 3 (t) = e yz , 0, 0 = 1, 0, 0
 
 
F 4 (t) = e yz , 0, 0 = et1 , 0, 0

Hence,
   
F 1 (t) 1 (t) = e1 , 0, 0 1, 0, 0 = e1
   
F 2 (t) 2 (t) = et1 , 0, 0 0, 1, 0 = 0
 
F 3 (t) 3 (t) = 1, 0, 0 1, 0, 0 = 1
   
F 4 (t) 4 (t) = et1 , 0, 0 0, 1, 0 = 0

We obtain the following integral:


 4 
  1  1
F ds = F ds = e1 dt + 0 + (1) dt + 0 = e1 1
C i=1 Ci 0 0

Step 2. Compute the curl.


 
 i j k 
 
 

 
curl(F) =   = e yz j e yz k = 0, e yz , e yz
 x y z 
 yz 
 e 0 0 

Step 3. Compute the flux of the curl through the surface. We parametrize the surface by

(x, y) = (x, y, 1), 0 x, y 1

The upward pointing normal is n = 0, 0, 1. We express curl(F) in terms of the parameters x and y:
 
curl(F) ((x, y)) = 0, e y1 , e y1

Hence,
 
curl(F) n = 0, e y1 , e y1 0, 0, 1 = e y1

The surface integral is thus


   1 1  1 1

curl(F) dS = e y1 d A = e y1 d y d x = e y1 d y = e y1 
S D 0 0 0 0
S E C T I O N 18.2 Stokes Theorem (ET Section 17.2) 1225

= 1 + e1 = e1 1 (1)

We see that the integrals in (1) and (2) are equal.


8. F = y, 2x, x + z, the upper hemisphere x 2 + y 2 + z 2 = 1, z 0
SOLUTION
Step 1. Compute the integral around the boundary curve. The boundary curve is the unit circle oriented in the counter-
clockwise direction. We use the parametrization (t) = (cos t, sin t, 0). Then,
  
F (t)  (t) =  sin t, 2 cos t, cos t  sin t, cos t, 0 = sin2 t + 2 cos2 t = 1 + cos2 t

The line integral is


  2  2   
1 1 3
F ds = (1 + cos2 t) dt = 1+ + cos 2t dt = 2 = 3
C 0 0 2 2 2
Step 2. Compute the flux of the curl through the hemisphere. We find the curl:
 
 i j k 

 

curl(F) =   = 0, 1, 3
 x y z 
 
 y 2x x + z 

We parametrize the upper hemisphere by ( , ) = (cos sin , sin sin , cos ), where 0 2 , 0 2 . We
compute the normal to the surface:

 = ( sin sin , cos sin , 0)


 = (cos cos , sin cos , sin )
 
 i j k 
 

  =  sin sin cos sin 0 

 cos cos sin cos sin 
 
2 2 sin2 sin 2 cos2 sin 2
= cos sin , sin sin ,
2 2

Since n points upward, we take n =   . Thus,




n = sin cos sin , sin sin , cos


curl(F) n = 0, 1, 3 sin cos sin , sin sin , cos = sin sin2 + 3 cos sin

We obtain:
  /2  2
curl(F) ds = sin sin2 + 3 cos sin d d
S 0 0
 /2    
2 /2
= 2
sin d sin d + 6 cos sin d
0 0 0

sin2  /2
= 0 + 6 = 3 (1)
2  0

The integrals in (1) and (2) are equal. This verifies Stokes Theorem.

In Exercises 910, use Stokes Theorem to compute the flux of curl(F) through the given surface.

9. F = z, y, x, the hemisphere x 2 + y 2 + z 2 = 1, x 0


SOLUTION By Stokes Theorem,
 
F ds = curl (F) dS
C S
We compute the line integral. The boundary curve, which is the unit circle in the (y, z) plane, is parametrized by

(t) = (0, cos t, sin t), 0 t 2


1226 C H A P T E R 18 F U N D A M E N TA L TH E O R E M S O F V E C T O R A N A LY S I S (ET CHAPTER 17)

We compute the following values:


 
F (t) = z, y, x = sin t, cos t, 0
 (t) = 0, sin t, cos t
  1
F (t)  (t) = sin t, cos t, 0 0, sin t, cos t = sin t cos t = sin 2t
2
Hence,
  2  2 
  1 cos 2t 2
F ds = F (t)  (t) dt = sin 2t dt = =0
C 0 0 2 4 0

Then also

curl(F) dS = 0
S
 
10. F = x 2 + y 2 , x + z 2 , 0 , the part of the cone z 2 = x 2 + y 2 such that 2 z 4

SOLUTION

C1
4

2
C2

y
x

The boundary C of the surface consists of the circle C1 : x 2 + y 2 = 16 in the plane z = 4 oriented clockwise, and the
circle C2 : x 2 + y 2 = 4 in the plane z = 2 oriented counterclockwise. We use the parametrizations

C1 : 1 (t) = (4 cos t, 4 sin t, 4), t from 2 to 0


C2 : 2 (t) = (2 cos t, 2 sin t, 2), t from 0 to 2

We compute the following values:


   
F 1 (t) = x 2 + y 2 , x + z 2 , 0 = 16, 4 cos t + 16, 0

1 (t) = 4 sin t, 4 cos t, 0


 
F 1 (t) 1 (t) = 16, 4 cos t + 16, 0 4 sin t, 4 cos t, 0 = 64 sin t + 16 cos2 t + 64 cos t
   
F 2 (t) = x 2 + y 2 , x + z 2 , 0 = 4, 2 cos t + 4, 0

2 (t) = 2 sin t, 2 cos t, 0


 
F 2 (t) 2 (t) = 4, 2 cos t + 4, 0 2 sin t, 2 cos t, 0 = 8 sin t + 4 cos2 t + 8 cos t

We obtain the following integral:


  
F ds = F ds + F ds
C C1 C2
 0  2
= 64 sin t + 16 cos2 t + 64 cos t dt + 8 sin t + 4 cos2 t + 8 cos t dt
2 0
 2  2

= 12 cos2 t 56 cos t + 56 sin t dt = 6t 3 sin 2t  + 0 = 12
0 0

Using Stokes Theorem we have


 
curl(F) dS = F ds = 12
S C
S E C T I O N 18.2 Stokes Theorem (ET Section 17.2) 1227

11. Let S be the surface of the cylinder (not including the top and bottom) of radius 2 for 1 z 6, oriented with
outward-pointing normal (Figure 14).
(a) Indicate with an arrow the orientation of S (the top and bottom circles).


(b) Verify Stokes Theorem for S and F = yz 2 , 0, 0 .

x y
FIGURE 14

SOLUTION
(a) The induced orientation is defined so that as the normal vector travels along the boundary curve, the surface lies to
its left. Therefore, the boundary circles on top and bottom have opposite orientations, which are shown in the figure.
z

C1
6
n

C2
1

x y
 
(b) We verify Stokes Theorem for S and F = yz 2 , 0, 0 .
Step 1. Compute the integral around the boundary circles. We use the following parametrizations:

C1 : 1 (t) = (2 cos t, 2 sin t, 6), t from 2 to 0


C2 : 2 (t) = (2 cos t, 2 sin t, 1), t from 0 to 2

We compute the following values:


   
F 1 (t) = yz 2 , 0, 0 = 72 sin t, 0, 0 ,

1 (t) = 2 sin t, 2 cos t, 0


 
F 1 (t) 1 (t) = 72 sin t, 0, 0 2 sin t, 2 cos t, 0 = 144 sin2 t
   
F 2 (t) = yz 2 , 0, 0 = 2 sin t, 0, 0 ,

2 (t) = 2 sin t, 2 cos t, 0


 
F 2 (t) 2 (t) = 2 sin t, 0, 0 2 sin t, 2 cos t, 0 = 4 sin2 t

The line integral is thus


    0  2
F ds = F ds + F ds = (144 sin2 t) dt + (4 sin2 t) dt
C C1 C2 2 0
 2  2 
1 cos 2t 70 sin 2t 2
= 140 sin2 t dt = 140 dt = 70 2  = 140
0 0 2 2 0

Step 2. Compute the curl


 
 i j k 
 
   
 
curl(F) =   = (2yz)j z 2 k = 0, 2yz, z 2
 x y z 
 
 yz 2 0 0 

Step 3. Compute the flux of the curl through the surface. We parametrize S by

( , z) = (2 cos , 2 sin , z), 0 2 , 1z6


1228 C H A P T E R 18 F U N D A M E N TA L TH E O R E M S O F V E C T O R A N A LY S I S (ET CHAPTER 17)

In Example 2 of Chapter 17.4 it is shown that the outward pointing normal is

n = 2 cos , 2 sin , 0

We compute the dot product:


 
curl(F) (( , z)) n = 0, 4z sin , z 2 2 cos , 2 sin , 0 = 8z sin2

We obtain the following integral (and use the integral we computed before):
       6
6 2 6 2 
curl(F) dS = 8z sin2 d dz = 8z dz sin2 d = 4z 2  = 140
S 1 0 1 0 1

The line integral and the flux have the same value. This verifies Stokes Theorem.
12. Let S be the portion of the plane z = x contained in the half-cylinder of radius R depicted in Figure 15. Use
Stokes Theorem to calculate the circulation of F = z, x, y + 2z around the boundary of S (a half-ellipse) in the
counterclockwise direction when viewed from above. Hint: Show that curl(F) is orthogonal to the normal vector to the
plane.
z

y
x
FIGURE 15

SOLUTION By Stokes Theorem,


 
F ds = curl(F) dS
C S
We compute the curl:
 
 i j k 
 
 
 
curl(F) =   = i + j + k = 1, 1, 1
 x y z 
 
 z x y + 2z 

The outward pointing vector of the plane z = x, or x z = 0, is n = 1, 0, 1. Therefore,

curl(F) n = 1, 1, 1 1, 0, 1 = 1 + 0 + 1 = 0

We conclude that
 
curl(F) ds = 0dA = 0
S D
This equation, along with Stokes Theorem, allows us to conclude that the circulation of F around C is zero.

2
13. Let I be the flux of F = e y , 2xe x , z 2 through the upper hemisphere S of the unit sphere.

2
(a) Let G = e y , 2xe x , 0 . Find a vector field A such that curl(A) = G.
(b) Use Stokes Theorem to show that the flux of G through S is zero.
Hint: Calculate the circulation of A around S.
(c) Calculate I . Hint: Use (b) to show that I is equal to the flux of 0, 0, z 2 through S.
SOLUTION
(a) We search for a vector field A so that G = curl(A). That is,
   
A3 A2 A1 A3 A2 A1 2
, , = e y , 2xe x , 0
y z z x x y
We note that the third coordinate ofthis curl vector must be zero; this canbe satisfied if A1 = 0 and A2 = 0. With this
2 2
in mind, we let A = 0, 0, e y e x . The vector field A = 0, 0, e y e x satisfies this equality. Indeed,

A3 A2 A1 A3 2 A2 A1
= ey , = 2xe x , =0
y z z x x y
S E C T I O N 18.2 Stokes Theorem (ET Section 17.2) 1229
 2

(b) We found that G = curl(A), where A = 0, 0, e y e x . We compute the flux of G through S. By Stokes Theorem,
  
G dS = curl(A) dS = A ds
S S C

The boundary C is the circle x 2 + y 2 = 1, parametrized by

(t) = (cos t, sin t, 0), 0 t 2


z

y
C
x

We compute the following values:


   2
  2

A (t) = 0, 0, e y e x = 0, 0, esin t ecos t

 (t) =  sin t, cos t, 0


   
A (t)  (t) = 0, 0, esin t ecos t  sin t, cos t, 0 = 0
2

Therefore,
  2
A ds = 0 dt = 0
C 0
 2

(c) We rewrite the vector field F = e y , 2xe x , z 2 as
 2
  2
    
F = e y , 2xe x , z 2 = e y , 2xe x , 0 + 0, 0, z 2 = curl(A) + 0, 0, z 2

Therefore,
    
F dS = curl(A) dS + 0, 0, z 2 dS (1)
S S S
In part (b) we showed that the first integral on the right-hand side is zero. Therefore,
   
F dS = 0, 0, z 2 dS (2)
S S
The upper hemisphere is parametrized by

( , ) = (cos sin , sin sin , cos ), 0 2 , 0 .
2
with the outward pointing normal


n = sin cos sin , sin sin , cos

See Example 4, Section 17.4. We have


 
0, 0, cos2 n = sin cos3

Therefore,
    2  /2  /2
0, 0, z 2 dS = sin cos3 d d = 2 sin cos3 d
S 0 0 0

cos4  /2 
= 2  = (0 1) =
4 0 2 2
Combining with (2) we obtain the solution


F dS = .
S 2
1230 C H A P T E R 18 F U N D A M E N TA L TH E O R E M S O F V E C T O R A N A LY S I S (ET CHAPTER 17)

14. Let F = 0, z, 1 and let S be the spherical cap x 2 + y 2 + z 2 1, where z 12 . Evaluate F dS directly
S
as a surface integral. Then verify that F = curl(A), where A = (0, x, x z) and evaluate the surface integral again using
Stokes Theorem.
SOLUTION

z
1

1
2
C
f0

x y

We first compute the surface integral directly. The spherical cap is parametrized by

( , ) = (cos sin , sin sin , cos ), 0 2 , 0
3

1
2 1
f0

cos f 0 = 1
2
f 0 = p3

The outward pointing normal is

n = sin (cos sin , sin sin , cos )

See Example 4, Section 17.4. Hence,


 

F ( , ) n = 0, cos , 1 n = sin sin2 cos + sin cos

We obtain the following integral:


  2  /3
F dS = sin sin2 cos + sin cos d d
S 0 0
     /3
2 /3
= sin d sin2 cos d + 2 sin cos d
0 0 0
 /3    /3
cos 2  3
=0+ sin 2 d =  = (1)
2  4
0 0

We now evaluate the flux using Stokes Theorem. We first notice that F = curl(A), where A = 0, x, x z. We verify it:
A3 A2
=00=0
y z
A1 A3
= 0 z = z
z x
A2 A1
=10=1
x y
Indeed, we see that curl(A) = F. Applying Stokes Theorem, we have
  
F dS = curl(A) dS = A ds (2)
S S C

3
2
C 1 f0 1
2

x y
S E C T I O N 18.2 Stokes Theorem (ET Section 17.2) 1231

To compute the line integral, we notice that the boundary curve is the circle x 2 + y 2 = 34 in the plane z = 12 . We
parametrize C by
 
3 3 1
(t) = cos t, sin t, , 0 t 2
2 2 2

We compute the following values:


 
  3 3
A (t) = 0, x, x z = 0, cos t, cos t
2 4
 
 3 3
(t) = sin t, cos t, 0
2 2
   
   3 3 3 3 3
A (t) (t) = 0, cos t, cos t sin t, cos t, 0 = cos2 t
2 4 2 2 4

We obtain the following line integral:


  2  2    2
3 2 3 3 3t 3  3
A ds = cos t dt = + cos 2t dt = + sin 2t  = (3)
C 0 4 0 8 8 8 16 0 4
Combining with (2), we obtain

3
F dS =
S 4
This values agrees with the solution obtained in (1), as expected.
15. Let A be the vector potential and B the magnetic field of the infinite solenoid of radius R in Example 5. Use Stokes
Theorem to compute:
(a) The flux of B through a circle in the x y-plane of radius r < R
(b) The circulation of A around the boundary C of a surface lying outside the solenoid
SOLUTION
(a) In Example 5 it is shown that B = curl(A), where
 

1 y x
R2 B 2 , 2 , 0 if r>R
2 r r
A= (1)


1 B y, x, 0 if r<R
2
Therefore, using Stokes Theorem, we have (S is the disk of radius r in the x y-plane)
  
B dS = curl(A) dS = A ds (2)
S S S

S
x
r

We parametrize the circle C = S by c(t) = r cos t, r sin t, 0, 0 t 2 . Then

c (t) = r sin t, r cos t, 0

By (1) for r < R,


1
A (c(t)) = B r sin t, r cos t, 0
2
Hence,
1 1 1
A (c(t)) c (t) = B r sin t, r cos t, 0 r sin t, r cos t, 0 = B r 2 sin2 t + r 2 cos2 t = r 2 B
2 2 2
1232 C H A P T E R 18 F U N D A M E N TA L TH E O R E M S O F V E C T O R A N A LY S I S (ET CHAPTER 17)

Now, by (2) we get


   2  2
1 2 1
B dS = A dS = r B dt = r 2 B dt = r 2 B
S S 0 2 2 0

(b) Outside the solenoid B is the zero field, hence B = 0 on every domain lying outside the solenoid. Therefore, Stokes
Theorem implies that
   
A dS = curl(A) dS = B dS = 0 dS = 0.
S S S S

16. The magnetic field B due to a small current loop (which we place at the origin) is called a magnetic dipole (Figure
16). Let = (x 2 + y 2 + z 2 )1/2 . For large, B = curl(A), where
 
y x
A = 3, 3,0

(a) Let C be a horizontal circle of radius R located far from the origin with center on z-axis. Show that A is tangent to C.
(b) Use Stokes Theorem to calculate the flux of B through C.

y
x Current loop

FIGURE 16

SOLUTION
(a) We parametrize C by

c(t) = (R cos t, R sin t, z 0 ) , 0 t 2

Then, the tangent to c(t) is in the direction of

c (t) = R sin t, R cos t, 0


z

A
z0
R

y
x Current loop
 
We write A = y3 , x3 , 0 in terms of the parameter t:

 
R sin t R cos t
A (c(t)) = 3 , , 0
3

A (c(t)) = 13 c (t). Therefore, A is parallel to c (t). We conclude that A is tangent to C.



(b) Let S be the disc enclosed in C. By Stokes Theorem the flux of B through S is
  
B dS = curl(A) dS = A ds
S S C

We compute the line integral. Since A and c (t) are parallel, we have
 

   R 2 sin2 t R 2 cos2 t

A (c(t)) c (t) = A (c(t)) c (t) =  + R 2 sin2 t + R 2 cos2 t
6 6
R R2 R2
= 3 R= 3 = 3/2

R 2 cos2 t + R 2 sin2 t + z 02
S E C T I O N 18.2 Stokes Theorem (ET Section 17.2) 1233

R2 1
= =
 z 0 2 3/2  z 0 2 3/2
R 2 cos2 t + sin2 t + R R 1+ R

Hence,
   2
dt 2
B dS = A ds = =
S C 0  z 0 2 3/2  z 0 2 3/2
R 1+ R R 1+ R

17. A uniform magnetic field B has constant strength b teslas in the z-direction [i.e., B = 0, 0, b].
(a) Verify that A = 12 B r is a vector potential for B, where r = x, y, 0.
(b) Calculate the flux of B through the rectangle with vertices A, B, C, and D in Figure 17.

FIGURE 17

SOLUTION

(a) We compute the vector A = 12 B r. Since B = bk and r = xi + yj, we have


 
1 1 1 1 by bx
A= B r = bk (xi + yj) = b(xk i + yk j) = b(xj yi) = , ,0
2 2 2 2 2 2
We now show that curl(A) = B. We compute the curl of A:
 
 i j k 

   

curl (A) =  x  = 0, 0, b + b = 0, 0, b = B
y z 
  2 2
 by bx 
 0 
2 2
Therefore, A is a vector potential for B.
(b) By Stokes Theorem the flux of B through the surface S is
  
B dS = curl(A) dS = A ds (1)
S S C
We compute the line integral. The unit circle C is parametrized by

c(t) = cos t, sin t, 0 , 0 t < 2


 
In part (a) we found that A = by bx
2 , 2 , 0 . Hence,
 
b sin t b cos t b b b 2 b
A (c(t)) c (t) = , , 0  sin t, cos t, 0 = sin2 t + cos2 t + 0 = sin t + cos2 t =
2 2 2 2 2 2
We obtain the following line integral:
  2
b b
A ds = dt = 2 = b (2)
C 0 2 2
We combine (1) and (2) to conclude that the flux of B through S is

B dS = b
S

2
 Let F = x y, x, 0 . Referring to Figure 17, let C be the closed path ABC D. Use Stokes Theorem to evaluate
18.
F ds in two ways. First, regard C as the boundary of the rectangle with vertices A, B, C, and D. Then treat C as the
C
boundary of the wedge-shaped box with open top.
1234 C H A P T E R 18 F U N D A M E N TA L TH E O R E M S O F V E C T O R A N A LY S I S (ET CHAPTER 17)

SOLUTION Let S1 be the rectangle whose boundary is C, and let S2 denote the wedge-shaped box. Then by Stokes
Theorem,
 
F ds = curl(F) dS (1)
C S1

and:
 
F ds = curl(F) dS (2)
C S2
 
We find the curl of F = x 2 y, x, 0 :
 
 i j k 
 
   
 
curl(F) =   = 0, 0, 1 + x 2
 x y z 
 
 x 2 y x 0 

We first compute the line integral via the surface integral in (1). To find a parametrization for S1 , we compute the equation
of the plane through A = (6, 0, 4), C = (0, 3, 0), D = (0, 0, 4). A normal to the plane is

AD AC = 6, 0, 0 6, 3, 4 = 6i (6i + 3j 4k) = 18k 24j
= 0, 24, 18 = 6 0, 4, 3

We use the point-normal equation of the plane:

0(x 0) + 4(y 3) + 3(z 0) = 0


4
4y 12 + 3z = 0 z =4 y
3
We parametrize S1 by
 
4y
(x, y) = x, y, 4
3
With the parameter domain, D = [0, 6] [0, 3] in the x y-plane.
y

x
0 6

Then
   
  4 4 4 4
= 1, 0, 0 0, 1, = i (j k) = k + j = 0, , 1
x y 3 3 3 3
The upward pointing normal is
 
4
n = 0, , 1
3
Also,
 
curl(F) ((x, y)) = 0, 0, 1 + x 2

Hence,
   4 
curl(F) n = 0, 0, 1 + x 2 0, , 1 = 1 + x 2
3
The integral in (1) is thus
    3 6  6
F ds = curl(F) dS = (1 + x 2 ) d A = (1 + x 2 ) d x d y = 3 (1 + x 2 ) d x
C S1 D 0 0 0
S E C T I O N 18.2 Stokes Theorem (ET Section 17.2) 1235
  
x 3 6
=3 x+ = 234 (3)
3 0

We now compute the line integral via the surface integral (2). The surface S2 consists of two rectangles R1 and R2 and
two triangles T1 and T2 , parametrized by

R1 : 1 (x, z) = (x, 0, z), 0 x 6, 0z4


n 1 = 0, 1, 0
R2 : 2 (x, y) = (x, y, 0), 0 x 6, 0y3
n 2 = 0, 0, 1
T1 : 3 (y, z) = (6, y, z), (y, z) D3
n 3 = 1, 0, 0
z

D3

y
0 3

T2 : 4 (y, z) = (0, y, z), (y, z) D4


n 4 = 1, 0, 0
z

D4

y
0 3

We compute each one of the surface integrals.


 
curl(F) (1 (x, z)) n1 = 0, 0, 1 + x 2 0, 1, 0 = 0
 
curl(F) (2 (x, y)) n2 = 0, 0, 1 + x 2 0, 0, 1 = 1 + x 2
 
curl(F) (3 (x, y)) n3 = 0, 0, 1 + 62 1, 0, 0 = 0
 
curl(F) (4 (y, z)) n4 = 0, 0, 1 + 02 1, 0, 0 = 0

Therefore the only nonzero integral is through R2 . We obtain


 
F ds = curl(F) dS
C S2
   
= curl(F) dS + curl(F) dS + curl(F) dS + curl(F) dS
R1 R2 T1 T2
  3 6  6
= curl(F) dS = (1 + x 2 ) d x d y = 3 (1 + x 2 ) d x
R2 0 0 0
 
x3 6
=3 x+  = 234 (4)
3 
0

The values in (3) and (4) match as expected.




19. Let F = z 2 , 2zx, 4y x2 and let C be a simple closed curve in the plane x + y + z = 4 that encloses a region of
area 16 (Figure 18). Calculate F ds, where C is oriented in the counterclockwise direction (when viewed from above
C
the plane).
1236 C H A P T E R 18 F U N D A M E N TA L TH E O R E M S O F V E C T O R A N A LY S I S (ET CHAPTER 17)

x+y+z=4
4

4 y
4
x
FIGURE 18

SOLUTION We denote by S the region enclosed by C. Then by Stokes Theorem,


 
F ds = curl(F) ds (1)
c S
 
We compute the curl of F = z 2 , 2zx, 4y x 2 :
 
 i j k 
 
 
 
curl(F) =   = 4 2x, 2x 2z, 2z
 x y z 
 
 z 2 2zx 4y x 2 

The plane x + y + z = 4 has the parametrization

(x, y) = x, y, 4 x y

Hence,
 
= 1, 0, 1 0, 1, 1 = (i k)(j k) = k + j + i = 1, 1, 1
x y
The normal determined by the induced orientation is

n = 1, 1, 1

Let D be the parameter domain in the parametrization (x, y) = (x, y, 4 x y) of S; that is, D will be the base
triangle in the x y plane that lies underneath the pyramid in the picture. To compute the surface integral in (1) we compute
the values

curl(F) ((x, y)) = 4 2x, 2x 2(4 x y), 2(4 x y) = 4 2x, 8 + 4x + 2y, 8 2x 2y
curl(F) n = 4 2x, 8 + 4x + 2y, 8 2x 2y 1, 1, 1 = 4 2x 8 + 4x + 2y + 8 2x 2y = 4

Therefore, using (1) and (2) we obtain


  
16
F ds = curl(F) dS = 4 d A = 4Area(D) = 4 = 32
C S D 2


20. Let F = y 2 , x 2 , z 2 . Show that
 
F ds = F ds
C1 C2

for any two closed curves lying on a cylinder whose central axis is the z-axis (Figure 19).

C2

C1

y
x
FIGURE 19
S E C T I O N 18.2 Stokes Theorem (ET Section 17.2) 1237

SOLUTION We denote by S the part of the cylinder for which C1 and C2 are boundary curves. Using Stokes Theorem
(notice that C1 and C2 have the same orientations), we have
  
F ds F ds = curl(F) dS (1)
C1 C2 S

We compute the curl:


 
F3 F2 F1 F3 F2 F1
curl(F) = , , = 0, 0, 2x 2y
y z z x x y
We parametrize S by

( , z) = R cos , R sin , z

where ( , z) varies in a certain parameter domain D. The outward-pointing normal is

n = R cos , R sin , 0

We compute curl(F) in terms of the parameters:

curl(F) = 0, 0, 2x 2y = 0, 0, 2R cos 2R sin 

We compute the dot product:

curl(F) n = 2R 0, 0, cos sin  R cos , sin , 0 = 2R 2 (0 + 0 + 0) = 0

Combining with (1) gives


   
F ds F ds = curl(F) dS = 0 d dr = 0
C1 C2 S D
or
 
F ds = F ds.
C1 C2

x y z
21. Let C be the triangular boundary of the portion of the plane + + = 1 lying in the octant x, y, z 0. Use
a b c

Stokes Theorem to find positive constants a, b, c such that the line integral of F = y 2 , 2z + x, 2y 2 around C is zero.
Hint: Choose constants so that curl(F) is orthogonal to the normal vector.
SOLUTION

b y
a
x

We compute the curl:


 
 i jk 

 

curl(F) =   = (4y 2)i + (1 2y)k = 4y 2, 0, 1 2y
 x y
z 
 
 y2 2y 2 
2z + x
 
The outward-pointing normal to the plane is n = a1 , b1 , 1c . Hence,
 
1 1 1 4y 2 1 2y
curl(F) n = 4y 2, 0, 1 2y , , = +
a b c a c
Note that if we choose a = 2 and c = 1, then curl(F) n = 0 (here, b can be any positive number). Thus, using Stokes
Theorem, we have
 
F ds = curl(F) n d A = 0
C D
1238 C H A P T E R 18 F U N D A M E N TA L TH E O R E M S O F V E C T O R A N A LY S I S (ET CHAPTER 17)

origin is
v0 = 3, 1, 4. Estimate the circulation around the small parallelogram
22. The curl of a vector field
F at the
spanned by the vectors A = 0, 12 , 12 and B = 0, 0, 13 .
SOLUTION We use the following approximation, relying on Stokes Theorem:

F ds (v0 en ) A(P) (1)
C
z

(
A = 0, 12 , 1
2 )
(
B = 0, 0, 1
3 )
en
0 y
x

The unit normal vector in the positive x-direction is

en = 1, 0, 0 (2)
   

We compute the area of the parallelogram spanned by the vectors O A = 0, 12 , 12 and O B = 0, 0, 13 :

1 1 1 1
O A O B = (j + k) k = (j k + k k) = i
2 3 6 6
 
1  1
A(P) =  
 6 i = 6 (3)

We now substitute v0 = 3, 1, 4, (2), and (3) in (1) to obtain the approximation

1 1 1
F ds 3, 1, 4 1, 0, 0 = 3 =
C 6 6 2

23. You know two things about a vector field F:


(a) F has a vector potential A (but A is unknown).
(b) F(x, y, 0) = 0, 0, 1 for all (x, y).
Determine the flux of F through the surface S in Figure 20.

1
y
Unit circle
x
FIGURE 20

SOLUTION Since F has a vector potentialthat is, F is the curl of a vector fieldthe flux of F through a surface
depends only on the boundary curve C. Now, the surface S and the unit disc S1 in the x y-plane share the same boundary
C. Therefore,
 
F dS = F dS (1)
S S1

x
1
S E C T I O N 18.2 Stokes Theorem (ET Section 17.2) 1239

We compute the flux of F through S1 , using the parametrization


S1 : (r, ) = (r cos , r sin , 0), 0 r 1, 0 2
n = 0, 0, 1
By the given information, we have
F ((r, )) = F(r cos , r sin , 0) = 0, 0, 1
Hence,
F ((r, )) n = 0, 0, 1 0, 0, 1 = 1
We obtain the following integral:
  2  1  2  1
F dS = F ((r, )) n dr d = 1 dr d = 2
S1 0 0 0 0

Combining with (1) we obtain



F dS = 2
S

24. Find the flux of F through the surface S in Figure 20, assuming that F has a vector potential and F(x, y, 0) =
cos x, 0, 0.
SOLUTION Since F has a vector potentialthat is, F is the curl of a vector fieldthe flux of F through a surface
depends only on the boundary curve C. Now, the surface S and the unit disc S1 in the x y-plane share the same boundary
C. Therefore,
 
F dS = F dS (1)
S S1
z

S1
y
C
x

We compute the flux of F through S1 , using the parametrization


S1 : (r, ) = (r cos , r sin , 0), 0 r 1, 0 2
n = 0, 0, 1
By the given information, we have
F n = cos x, 0, 0 0, 0, 1 = 0
We obtain the following integral:
  2  1
F dS = F n dr d = 0
S1 0 0

25. Use Eq. (8) to prove that if a is a constant vector, then curl( a) = a.
SOLUTION By Eq. (8) we have
curl( a) = curl(a) + a
Since a is a constant vector, all the partial derivatives of the components of a are zero, hence the curl of a is the zero
vector:
curl(a) = 0
Thus we obtain
curl( a) = a
1240 C H A P T E R 18 F U N D A M E N TA L TH E O R E M S O F V E C T O R A N A LY S I S (ET CHAPTER 17)

26. A vector field F is called radial if it is of the form F = () x, y, z for some function (), where =

x 2 + y 2 + z 2 . Show that the curl of a radial vector field is zero. Hint: It is enough to show that one component of
the curl is zero, since the calculation for the other two components is similar by symmetry.
SOLUTION Let v = x, y, z. We must show that curl( v) = 0. By Eq. (8) we have

curl( v) = curl(v) + v

We compute the curl of v:


 
 i j k 
 
 
 
curl(v) =   = 0i + 0j + 0k = 0
 x y z 
 
 x y z 

Therefore,

curl( v) = v (1)

We now must prove that the cross product is zero. To compute the gradient of , we use the derivatives
2x x
=  =
x 2 x +y +z
2 2 2
2y y
=  =
y 2 x +y +z
2 2 2
2z z
=  =
z 2 x 2 + y2 + z2

By the Chain Rule we get


d d x
= =
x d x d
d d y
= =
y d y d
d d z
= =
z d z d
Therefore,
 
1 d 1 d
= , , = x, y, z = v
x y z d d
We see that is a scalar multiple of v, hence the cross product of the two vectors is the zero vector:

v = 0

Combining with (1) we get

curl( v) = 0

27. Verify the identity

curl(( )) = 0 7

SOLUTION We have
 

= , ,
x y z
 
We compute each component of curl ( ) . The first component is
   
2 2
= =0
y z z y yz zy
The second component of curl(( )) is
   
2 2
= =0
z x x z z x xz
S E C T I O N 18.2 Stokes Theorem (ET Section 17.2) 1241

And the third component is


   
2 2
= =0
x y y x xy y x
We conclude that

curl(( )) = 0

28. Prove the Product Rule

curl( F) = curl(F) + F 8


SOLUTION We evaluate the curl of F. Since F = F1 , F2 , F3 , using the Product Rule for scalar functions we
have
 

curl( F) = ( F3 ) ( F2 ), ( F1 ) ( F3 ), ( F2 ) ( F1 )
y z z x x y

F F F F
= F + 3 F 2, F + 1 F 3,
y 3 y z 2 z z 1 z x 3 x

F F
F + 2 F 1
x 2 x y 1 y
 
F3 F2 F1 F3 F2 F1
= , ,
y z z x x y
 

+ F3 F2 , F1 F3 , F2 F1 (1)
y z z x x y
The vector in the first term is curl(F). We show that the second term is the cross product F. We compute the cross
product:
 
 i j k 

  
 

 


 
F =   = F3 F2 i F3 F1 j + F2 F1 k
 x y z  y z x z x y
 
 F1 F2 F3 
 

= F3 F2 , F1 F3 , F2 F1
y z z x x y
Therefore,(1) gives

curl( F) = curl(F) + F

29. Assume that the second partial derivatives of and exist and are continuous. Use(7) and (8) to prove that
 
( ) ds = ( ) ( ) ds
S S
where S is a smooth surface with boundary S.
SOLUTION By Stokes Theorem, we have
 
( ) ds = curl( ) dS
S S
We now use Eq.(8) to evaluate the curl of . That is,
 
 
( ) ds = curl( ) + dS
S S
 
= curl( ) dS + ( ) ( ) dS (1)
S S
Now, since the gradient field is conservative, this field satisfies the cross-partials condition. In other words,

curl( ) = 0

Combining with(1) we obtain


   
( ) ds = 0 dS + ( ) ( ) dS = ( ) ( ) dS
S S S S
1242 C H A P T E R 18 F U N D A M E N TA L TH E O R E M S O F V E C T O R A N A LY S I S (ET CHAPTER 17)

30. Explain carefully why Greens Theorem is a special case of Stokes Theorem.
SOLUTION Let C be a simple closed curve enclosing a region D oriented counterclockwise in the x y-plane. We must
show, using Stokes Theorem, that for F = F1 , F2 ,
   
F2 F1
F ds = dA
C D x y
We consider D as a surface in three-space with parametrization

(x, y) = (x, y, 0), (x, y) D

The normal vector is

n = 0, 0, 1
z

D y

x C

We compute the curl of F = F1 , F2 , 0. Since F1 = F1 (x, y), F2 = F2 (x, y), and F3 = 0, the curl of F is the vector
   
F3 F2 F1 F3 F2 F1 F F1
curl(F) = , , = 0, 0, 2
y z z x x y x y
Hence,
 
F F1 F2 F1
curl(F) n = 0, 0, 2 0, 0, 1 =
x y x y
By Stokes Theorem we have
     
F2 F1
F ds = curl(F) dS = curl(F) n d A = dA
C D D D x y
We thus showed that Greens Theorem is a special case of Stokes Theorem for two dimensions.

Further Insights and Challenges


31. Complete the proof of Theorem 1 by proving the equality
 
F3 (x, y, z)k ds = curl(F3 (x, y, z)k) dS
C S
where S is the graph of a function z = f (x, y) over a domain D in the x y-plane whose boundary is a simple closed
curve.
SOLUTION Let (x(t), y(t)), a t b be a parametrization of the boundary curve C0 of the domain D.

(x, y, f(x, y))


n

(x, y) y
D C0
x
S E C T I O N 18.2 Stokes Theorem (ET Section 17.2) 1243

The boundary curve C of S projects on C0 and has the parametrization

(t) = (x(t), y(t), f (x(t), y(t))) , at b

Let

F = 0, 0, F3 (x, y, z)

We must show that


 
F ds = curl(F) dS (1)
C S
We first compute the surface integral, using the parametrization

S : (x, y) = (x, y, f (x, y))

The normal vector is


 
 
n= = 1, 0, f x (x, y) 0, 1, f y (x, y) = (i + f x (x, y)k) j + f y (x, y)k
x y


= f y (x, y)j f x (x, y)i + k = f x (x, y), f y (x, y), 1

We compute the curl of F:


 
 i j k 
   
  F3 (x, y, z) F (x, y, z)
 
curl(F) =  = , 3 ,0
 x y z  y x
 
 0 0 F3 (x, y, z) 

Hence,
 
F3 F3

curl(F) ((x, y)) n = (x, y, f (x, y)) (x, y, f (x, y)) , 0 f x (x, y), f y (x, y), 1
y x
F3 (x, y, f (x, y)) F3 (x, y, f (x, y))
= f x (x, y) + f y (x, y)
y x
The surface integral is thus
   
F3 (x, y, f (x, y)) F3 (x, y, f (x, y))
curl(F) dS = f x (x, y) + f y (x, y) d x d y (2)
S D y x
We now evaluate the line integral in (1). We have
  
     d  
F (t)  (t) = 0, 0, F3 x(t), y(t), f x(t), y(t) x  (t), y  (t), f x(t), y(t)
dt

 d  
= F3 x(t), y(t), f x(t), y(t) f x(t), y(t) (3)
dt
Using the Chain Rule gives
d      
f x(t), y(t) = f x x(t), y(t) x  (t) + f y x(t), y(t) y  (t)
dt
Substituting in (3), we conclude that the line integral is
  b 
     
F ds = F3 x(t), y(t), f x(t), y(t) f x x(t), y(t) x  (t) + f y x(t), y(t) y  (t) dt (4)
C a

We consider the following vector field:



   
G(x, y) = F3 x, y, f (x, y) f x (x, y), F3 x, y, f (x, y) f y (x, y)

Then the integral in (4) is the line integral of the planar vector field G over C0 . That is,
 
F ds = G ds
C C0

Therefore, we may apply Greens Theorem and write


    
   
F ds = G ds = F3 x, y, f (x, y) f y (x, y) F3 x, y, f (x, y) f x (x, y) d x d y (5)
C C0 D x y
1244 C H A P T E R 18 F U N D A M E N TA L TH E O R E M S O F V E C T O R A N A LY S I S (ET CHAPTER 17)

We use the Product Rule to evaluate the integrand:


F3   F3    
(x, y, f (x, y)) f y (x, y) + F3 x, y, f (x, y) f yx (x, y) x, y, f (x, y) f x (x, y) F3 x, y, f (x, y) f x y (x, y)
x y
F3   F3  
= x, y, f (x, y) f y (x, y) x, y, f (x, y) f x (x, y)
x y
Substituting in (5) gives
   
F3 (x, y, f (x, y)) F3 (x, y, f (x, y))
F ds = f y (x, y) f x (x, y) dx dy (6)
C D x y
Equations(2) and(6) give the same result, hence
 
F ds = curl(F) ds
C S
for

F = 0, 0, F3 (x, y, z)

32. Let F be a continuously differentiable vector field in R3 , Q a point, and S a plane containing Q with unit normal
vector e. Let Cr be a circle of radius r centered at Q in S and let Sr be the disk enclosed by Cr . Assume Sr is oriented
with unit normal vector e.
(a) Let m(r ) and M(r ) be the minimum and maximum values of curl(F(P)) e for P Sr . Prove that

1
m(r ) 2 curl(F) dS M(r )
r Sr
(b) Prove that

1
curl(F(Q)) e = lim F ds
r 0 r 2 Cr

This proves that curl(F(Q)) e is the circulation per unit area in the plane S.
SOLUTION
(a) We may assume that the circle lies on the x y-plane, and parametrize Sr by

Sr : (x, y, z) = (x, y, 0), (x, y) Sr


Cr

Q
en(Q) = e

Sr

Then, n is a unit vector and we have


 
curl(F) dS = curl(F)(P) en (P) d A (1)
Sr Sr
We use the given information m(r ) curl(F)(P) en (P) M(r ) for P Sr and properties of the double integral to
write

A(Sr )m(r ) curl(F)(P) en (P) d A A(Sr ) M(r )
Sr

The area of the disk is A(Sr ) = r 2 . Therefore,



r 2 m(r ) curl(F)(P) en (P) d A r 2 M(r )
Sr
or

1
m(r ) curl(F)(P) en (P) d A M(r )
r 2 Sr
Combining with(1) we get

1
m(r ) curl(F) dS M(r )
r 2 Sr
S E C T I O N 18.3 Divergence Theorem (ET Section 17.3) 1245

(b) By Stokes Theorem,


 
F ds = curl(F) dS
Cr Sr

By part(a) we have

1
m(r ) F ds M(r ) (2)
r 2 Cr
We take the limit over the circles of radius r centered at Q, as r 0. As r 0, the regions Sr are approaching the
center Q. The continuity of the curl implies that

lim m(r ) = lim M(r ) = curl(F)(Q) en (Q) = curl(F)(Q) e


r 0 r 0

Therefore,

1
lim m(r ) lim F ds lim M(r )
r 0 r 0 r 2 Cr r 0

1
curl(F)(Q) e lim F ds curl(F)(Q) e
r 0 r 2 Cr

Hence,

1
lim F ds = curl(F)(Q) e
r 0 r 2 Cr

18.3 Divergence Theorem (ET Section 17.3)


Preliminary Questions
1. What is the flux of F = 1, 0, 0 through a closed surface?
SOLUTION The divergence of F = 1, 0, 0 is div(F) = Px + Q R
y + z = 0, therefore the Divergence Theorem
implies that the flux of F through a closed surface S is
  
F dS = div(F) d V = 0 dV = 0
S W W


2. Justify the following statement: The flux of F = x 3 , y 3 , z 3 through every closed surface is positive.
 
SOLUTION The divergence of F = x 3 , y 3 , z 3 is

div(F) = 3x 2 + 3y 2 + 3z 2

Therefore, by the Divergence Theorem, the flux of F through a closed surface S is


 
F dS = (3x 2 + 3y 2 + 3z 2 ) d V
S W
Since the integrand is positive for all (x, y, z)  = (0, 0, 0), the triple integral, hence also the flux, is positive.
3. Which of the following expressions are meaningful (where F is a vector field and is a function)? Of those that are
meaningful, which are automatically zero?
(a) div( ) (b) curl( ) (c) curl( )
(d) div(curl(F)) (e) curl(div(F)) (f) (div(F))
SOLUTION
(a) The divergence is defined on vector fields. The gradient is a vector field, hence div( ) is defined. It is not automat-
ically zero since for = x 2 + y 2 + z 2 we have

div( ) = div 2x, 2y, 2z = 2 + 2 + 2 = 6  = 0

(b) The curl acts on vector valued functions, and is such a function. Therefore, curl( ) is defined. Since the
gradient field is conservative, the cross partials of are equal, or equivalently, curl( ) is the zero vector.
(c) The curl is defined on vector fields rather than on scalar functions. Therefore, curl( ) is undefined. Obviously,
curl( ) is also undefined.
1246 C H A P T E R 18 F U N D A M E N TA L TH E O R E M S O F V E C T O R A N A LY S I S (ET CHAPTER 17)

(d) The curl is defined on the vector field F and the divergence is defined on the vector field curl(F). Therefore the
expression div (curl(F)) is meaningful. We show that this vector is automatically zero:
 
F3 F2 F1 F3 F2 F1
div (curl (F)) = div , ,
y z z x x y
     
F3 F2 F1 F3 F2 F1
= + +
x y z y z x z x y
2 F3 2 F2 2 F1 2 F3 2 F2 2 F1
= + +
xy xz yz y x z x zy
     
2
F3 2
F3 2
F2 2
F2 2 F1 2 F1
= + +
xy y x z x xz yz zy
=0+0+0=0

(e) The curl acts on vector valued functions, whereas div(F) is a scalar function. Therefore the expression curl (div(F))
has no meaning.
(f) div(F) is a scalar function, hence (divF) is meaningful. It is not necessarily the zero vector as shown in the follow-
ing example:
 
F = x 2, y2, z2

div (F) = 2x + 2y + 2z
(divF) = 2, 2, 2  = 0, 0, 0

4. Which of the following statements is correct (where F is a continuously differentiable vector field defined every-
where)?
(a) The flux of curl(F) through all surfaces is zero.
(b) If F = , then the flux of F through all surfaces is zero.
(c) The flux of curl(F) through all closed surfaces is zero.
SOLUTION

(a) This statement holds only for conservative
fields. If F is not conservative, there exist closed curves such that C F
ds  = 0, hence by Stokes Theorem S curl(F) dS  = 0.
(b) This statement is false. Consider the unit sphere S in the three-dimensional space and the function (x, y, z) =
x 2 + y 2 + z 2 . Then F = = 2x, 2y, 2z and div (F) = 2 + 2 + 2 = 6. Using the Divergence Theorem, we have (W
is the unit ball in R 3 )
   
F dS = div(F) d V = 6 dV = 6 d V = 6 Vol(W)
S W W W
(c) This statement is correct, as stated in the corollary of Stokes Theorem in section 18.2.


5. How does the Divergence Theorem imply that the flux of F = x 2 , y e z , y 2zx through a closed surface is equal
to the enclosed volume?
SOLUTION By the Divergence Theorem, the flux is
   
F dS = div(F) d V = (2x + 1 2x) d V = 1 d V = Volume(W)
S W W W
Therefore the statement is true.

Exercises
In Exercises 14, compute the divergence of the vector field.


1. F = x y, yz, y 2 x 3
SOLUTION The divergence of F is


div(F) = (x y) + (yz) + (y 2 x 3 ) = y + z + 0 = y + z
x y z

2. xi + yj + zk
S E C T I O N 18.3 Divergence Theorem (ET Section 17.3) 1247

SOLUTION


div(F) = (x) + (y) + (z) = 1 + 1 + 1 = 3
x y z


3. F = x 2zx 2 , z x y, z 2 x 2
SOLUTION


div(F) = (x 2zx 2 ) + (z x y) + (z 2 x 2 ) = (1 4zx) + (x) + (2zx 2 ) = 1 4zx x + 2zx 2
x y z

4. sin(x + z)i ye x z k
SOLUTION


div(F) = sin(x + z) + (ye x z ) = cos(x + z) yxe x z
x z

In Exercises 58, verify the Divergence Theorem for the vector field and region.

5. F = z, x, y and the box [0, 4] [0, 2] [0, 3]


SOLUTION Let S be the surface of the box and R the region enclosed by S.

2 y

We first compute the surface integral in the Divergence Theorem:


 
F dS = div(F) d V (1)
S R
We denote by Si , i = 1, . . . , 6, the faces of the box, starting at the face on the x z-plane and moving counterclockwise,
then moving to the bottom and the top. We use parametrizations

S1 : 1 (x, z) = (x, 0, z), 0 x 4, 0z3


n = 0, 1, 0
S2 : 2 (y, z) = (0, y, z), 0 y 2, 0z3
n = 1, 0, 0
S3 : 3 (x, z) = (x, 2, z), 0 x 4, 0z3
n = 0, 1, 0
S4 : 4 (y, z) = (4, y, z), 0 y 2, 0z3
n = 1, 0, 0
S5 : 5 (x, y) = (x, y, 0), 0 x 4, 0y2
n = 0, 0, 1
S6 : 6 (x, y) = (x, y, 3), 0 x 4, 0y2
n = 0, 0, 1

Then,
  3 4  3 4
F dS = F (1 (x, z)) 0, 1, 0 d x dz = z, x, 0 0, 1, 0 d x dz
S1 0 0 0 0
1248 C H A P T E R 18 F U N D A M E N TA L TH E O R E M S O F V E C T O R A N A LY S I S (ET CHAPTER 17)

 3 4 
x 2 4
= x d x dz = 3 = 24
0 0 2 0
  3 2  3 2
F dS = F (2 (y, z)) 1, 0, 0 d y dz = z, 0, y 1, 0, 0 d y dz
S2 0 0 0 0
 3 2 
z 2 3
= z d y dz = 2 = 9
0 0 2 0
  3 4  3 4
F dS = F (3 (x, z)) 0, 1, 0 d x dz = z, x, 2 0, 1, 0 d x dz
S3 0 0 0 0
 3 4 
x 2 4
= x d x dz = 3 = 24
0 0 2 0
  3 2  3 2
F dS = F (4 (y, z)) 1, 0, 0 d y dz = z, 4, y 1, 0, 0 d y dz
S4 0 0 0 0
 3 2 
z 2 3
= z d y dz = 2 =9
0 0 2 0
  2 4  2 4
F dS = F (5 (x, y)) 0, 0, 1 d x d y = 0, x, y 0, 0, 1 d x d y
S5 0 0 0 0
 2 4 
y 2 2
= y d x d y = 4 = 8
0 0 2 0
  2 4  2 4
F dS = F (6 (x, y)) n d x d y = 3, x, y 0, 0, 1 d x d y
S6 0 0 0 0
 2 4 
y 2 2
= y dx dy = 4 =8
0 0 2 0

We add the integrals to obtain the surface integral


 6 

F dS = F dS = 24 9 + 24 + 9 8 + 8 = 0 (2)
S i=1 Si

We now evaluate the triple integral in (1). We compute the divergence of F = z, x, y:

div(F) = (z) + (x) + (y) = 0
x y z
Hence,
 
div(F) d V = 0 dV = 0 (3)
R R
The equality of the integrals in (2) and (3) verifies the Divergence Theorem.
6. F = y, x, z and the region x 2 + y 2 + z 2 4
SOLUTION Let S be the surface of the sphere and R the ball enclosed by S. We compute both sides of the Divergence
Theorem:
 
F dS = div(F) d V (1)
S R
Step 1. Integral over sphere. We use the parametrization

S : ( , ) = (2 cos sin , 2 sin sin , 2 cos ), 0 2 , 0




n = 4 sin cos sin , sin sin , cos

Then,
 


F ( , ) n = 2 sin sin , cos sin , cos 4 sin cos sin , sin sin , cos

= 8 sin cos sin3 + cos sin sin3 + cos2 sin

= 8 sin 2 sin3 + 8 cos2 sin


S E C T I O N 18.3 Divergence Theorem (ET Section 17.3) 1249

The surface integral is thus


   2   2
 
F dS = F ( , ) n d d = 8 sin 2 sin3 + 8 cos2 sin d d
S 0 0 0 0
     
2
= 8 sin 2 d sin3 d + 16 cos2 sin d
0 0 0

 
cos3  16 32
= 0 + 16 = (1 1) =
3 0 3 3

We compute the triple integral in (1):



divF = div y, x, z = (y) + (x) + (z) = 1
x y z
 
4 23 32
div(F) d V = 1 d V = Volume(R) = = (2)
R R 3 3
The equality of the integrals in (2) and (3) verifies the Divergence Theorem.
7. F = 2x, 3z, 3y and the region x 2 + y 2 1, 0 z 2
SOLUTION

y
1

Let S be the surface of the cylinder and R the region enclosed by S. We compute the two sides of the Divergence
Theorem:
 
F dS = div(F) d V (1)
S R
We first calculate the surface integral.
Step 1. Integral over the side of the cylinder. The side of the cylinder is parametrized by

( , z) = (cos , sin , z), 0 2 , 0z2


n = cos , sin , 0

Then,

F (( , z)) n = 2 cos , 3z, 3 sin  cos , sin , 0 = 2 cos2 + 3z sin

We obtain the integral


  2  2  2    
2 2
F dS = 2 cos2 + 3z sin d dz = 4 cos2 d + 3z dz sin d
side 0 0 0 0 0
  
sin 2 2
=4 + + 0 = 4
2 4 0

Step 2. Integral over the top of the cylinder. The top of the cylinder is parametrized by

(x, y) = (x, y, 2)
!
with parameter domain D = (x, y) : x 2 + y 2 1 . The upward pointing normal is

n = Tx T y = 1, 0, 0 0, 1, 0 = i j = k = 0, 0, 1


1250 C H A P T E R 18 F U N D A M E N TA L TH E O R E M S O F V E C T O R A N A LY S I S (ET CHAPTER 17)

Also,

F ((x, y)) n = 2x, 6, 3y 0, 0, 1 = 3y

Hence,
 
F dS = 3y d A = 0
top D

The last integral is zero due to symmetry.


y

D
x
1

Step 3. Integral over the bottom of the cylinder. We parametrize the bottom by

(x, y) = (x, y, 0), (x, y) D

The downward pointing normal is n = 0, 0, 1. Then

F ((x, y)) n = 2x, 0, 3y 0, 0, 1 = 3y

We obtain the following integral, which is zero due to symmetry:


 
F dS = 3y d A = 0
bottom D
Adding the integrals we get
   
F dS = F dS + F dS + F dS = 4 + 0 + 0 = 4 (2)
S side top bottom

Step 4. Compare with integral of divergence.



div(F) = div 2x, 3z, 3y = (2x) + (3z) + (3y) = 2
x y z
  
div (F) d V = 2 dV = 2 d V = 2 Vol(R) = 2 2 = 4 (3)
R R R
The equality of (2) and (3) verifies the Divergence Theorem.

8. F = x, 0, 0 and the region x 2 + y 2 z 4


SOLUTION

Let S be the surface enclosing the given region R. We must verify the equality
 
F dS = div(F) d V (1)
S R
We first compute the surface integral on the left-hand side.
Step 1. Integral over the side of the surface. The side of the surface is parametrized by

( , t) = t cos , t sin , t 2 , 0 t 2, 0 2
S E C T I O N 18.3 Divergence Theorem (ET Section 17.3) 1251

The outward pointing normal is


 
 i j k   
 
n = T Tt =  t sin t cos 0  = 2t 2 cos , 2t 2 sin , t

 cos sin 2t 

Also,
 
F (( , t)) n = t cos , 0, 0 2t 2 cos , 2t 2 sin , t = 2t 3 cos2

The surface integral over the side is


  2  2    
2 2
F dS = 2t 3 cos2 dt d = cos2 d 2t 3 dt
side 0 0 0 0
    
sin 2 2 t 4 2
= + = 8 (2)
2 4  =0 2 t=0

Step 2. Integral over the top of the surface.


! The top of the surface is parametrized by (x, y) = (x, y, 4) with parameter
domain D = (x, y) : x 2 + y 2 2 . The upward pointing normal vector is

n = Tx T y = 1, 0, 0 0, 1, 0 = i j = k = 0, 0, 1

Also,

F ((x, y)) n = x, 0, 0 0, 0, 1 = 0

Hence,
 
F dS = 0dA = 0 (3)
top D

Adding the surface integrals (2) and (3) we get


  
F dS = F dS + F dS = 8 + 0 = 8
S side top

Step 3. Compare with integral of divergence.


y

x
2
D

We compute the divergence:



div(F) = (x) + (0) + (0) = 1
x y z
We obtain the following triple integral:
     
4
div(F) d V = 1 dV = dz dx dy = 4 (x 2 + y 2 ) d x d y
R R D x 2 +y 2 D
 2  2  2   
r 4 2
= 2
(4 r )r dr d = 2 3 2
4r r dr = 2 2r 
0 0 0 4 0
= 2 4 = 8 (4)

The equality of (2) and (4) verifies the Divergence Theorem.



In Exercises 916, use the Divergence Theorem to evaluate the surface integral F dS.
S
1252 C H A P T E R 18 F U N D A M E N TA L TH E O R E M S O F V E C T O R A N A LY S I S (ET CHAPTER 17)

9. F = x, y, z, S is the sphere x 2 + y 2 + z 2 = 1.


SOLUTION We compute the divergence of F = x, y, z:


div(F) = (x) + (y) + (z) = 1 + 1 + 1 = 3
x y z
Using the Divergence Theorem and the volume of the sphere, we obtain
   
F dS = div(F) d V = 3 dV = 3 1 d V = 3 Volume (W)
S W W W
4
= 3 13 = 4
3

10. F = y, z, x, S is the sphere x 2 + y 2 + z 2 = 1.


SOLUTION We compute the divergence of F = y, z, x:


div(F) = (y) + (z) + (x) = 0 + 0 + 0 = 0
x y z
Hence, by the Divergence Theorem (W is the unit ball),
  
F dS = div(F) d V = 0 dV = 0
S W W


11. F = x 3 , 0, z 3 , S is the sphere x 2 + y 2 + z 2 = 4.
 
SOLUTION We compute the divergence of F = x 3 , 0, z 3 :

3
div(F) = (x ) + (0) + (z 3 ) = 3x 2 + 3z 2 = 3(x 2 + z 2 )
x y z
Using the Divergence Theorem we obtain (W is the region inside the sphere)
  
F dS = div(F) d V = 3(x 2 + z 2 ) d V
S W W
We convert the integral to spherical coordinates. We have

x 2 + z 2 = 2 cos2 sin2 + 2 cos2 = 2 cos2 sin2 + 2 (1 sin2 )


= 2 sin2 (1 cos2 ) + 2 = 2 sin2 sin2 + 2 = 2 (1 sin2 sin2 )

We obtain the following integral:


  2   2
F dS = 3 2 (1 sin2 sin2 ) 2 sin d d d
S 0 0 0
 2   2
=3 4 (sin sin3 sin2 )d d d
0 0 0
 2   2  2   2
=3 4 sin d d d 3 4 sin3 sin2 d d d
0 0 0 0 0 0
   2  
2
 
  2 
= 6 sin d 4 d 3 sin2 d sin3 d 4 d
0 0 0 0 0
           5 2 
 5 2 sin 2 2 sin2 cos 2  
= 6 cos  3 cos 
=0 5 =0 2 4  =0 3 3 =0 5 =0
32 4 32 256
= 12 3 =
5 3 5 5

12. F = x, y, z, S is the boundary of the unit cube 0 x, y, z 1.


SOLUTION Let W denote the box [0, 1] [0, 1] [0, 1]. By the Divergence Theorem,
 
F dS = div(F) d V
S W
S E C T I O N 18.3 Divergence Theorem (ET Section 17.3) 1253

We compute the divergence of F = x, y, z:



div(F) = (x) + (y) + (z) = 1 1 + 1 = 1
x y z
Hence,
 
F dS = 1 d V = Volume(W) = 13 = 1.
S W


13. F = x, y 2 , z + y , S is the boundary of the region contained in the cylinder x 2 + y 2 = 4 between the planes z = x
and z = 8.
SOLUTION Let W be the region enclosed by S.
z

x y

 
We compute the divergence of F = x, y 2 , z + y :

2
div(F) = (x) + (y ) + (z + y) = 1 + 2y + 1 = 2 + 2y.
x y z
By the Divergence Theorem we have
  
F dS = div(F) d V = (2 + 2y) d V
S W W

We compute the triple integral. Denoting by D the disk x 2 + y 2 4 in the x y-plane, we have
   8  8 

F dS = (2 + 2y) dz d x d y = (2 + 2y)z  dx dy = (2 + 2y)(8 x) d x d y
S D x D z=x D

We convert the integral to polar coordinates:


  2  2
F dS = (2 + 2r sin )(8 r cos )r dr d
S 0 0
 2  2
= 16r + 2r 2 (8 sin cos ) r 3 sin 2 dr d
0 0
 2 2
2 r4 
= 8r 2 + r 3 (8 sin cos ) sin 2  d
0 3 4 r =0
 2  
16
= 32 + (8 sin cos ) 4 sin 2 d
0 3
 2   2
128 16 2
= 64 + sin d cos d 4 sin 2 d = 64
3 0 3 0 0


2
14. F = x 2 z 2 , e z cos x, y 3 , S is the boundary of the region bounded by x + 2y + 4z = 12 and the coordinate
planes in the first octant.
 2

SOLUTION We compute the divergence of F = x 2 z 2 , e z cos x, y 3 :

2  z2 
div(F) = (x z 2 ) + e cos x + (y 3 ) = 2x.
x y z
By the Divergence Theorem,
  
F dS = div(F) d V = 2x d V .
S W W
1254 C H A P T E R 18 F U N D A M E N TA L TH E O R E M S O F V E C T O R A N A LY S I S (ET CHAPTER 17)

6
y

12

To compute the triple integral, we describe the region W by the inequalities


x y x
0 x 12, 0y + 6, 0 z 3 .
2 2 4
y

6
y = 12 x + 6

x
12

Thus,
  12  x/2+6  3 y x  12  x/2+6 3 y x
2 4  2 4
F dS = 2x dz d y d x = 2x z  dy dx
S 0 0 0 0 0 z=0
 12  x/2+6
 12  
y x y2 x y x/2+6
= 2x 3 dy dx = 2x 3y dx
0 0 2 4 0 4 4  y=0
 12   2 
x x 6 x2
= 2x 3 6 dx
0 4 2 4

We let u = 6 x2 and du = 12 d x:
 12  2  6 
6 x2 2 3 1 4 6
2x dx = 2(6 u)u du = 4u u  = 216.
0 4 0 2 0

15. F = x + y, z, z x, S is the boundary of the region between the paraboloid z = 9 x 2 y 2 and the x y-plane.
SOLUTION We compute the divergence of F = x + y, z, z x,


div(F) = (x + y) + (z) + (z x) = 1 + 0 + 1 = 2.
x y z
z
9

y
3

Using the Divergence Theorem we have


  
F dS = div(F) d V = 2 dV
S W W
We compute the triple integral:
    9x 2 y 2  9x 2 y 2

F dS = 2 dV = 2 dz d x d y = 2z  dx dy
S W D 0 D 0

= 2(9 x 2 y 2 ) d x d y
W
S E C T I O N 18.3 Divergence Theorem (ET Section 17.3) 1255

x 2 + y2 = 9

x
3

We convert the integral to polar coordinates:

x = r cos , y = r sin , 0 r 3, 0 2
  2  3  3   
9r 2 r 4 3
F dS = 2 9 r 2 r dr d = 4 (9r r 3 ) dr = 4  = 81
S 0 0 0 2 4 0


2 
16. F = e z , sin(x 2 z), x 2 + 9y 2 , S is the region x 2 + y 2 z 8 x 2 y 2 .
SOLUTION We compute the divergence of F:

z2
div(F) = (e ) + sin(x 2 z) + x 2 + y2 = 0
x y z
The Divergence Theorem implies that
 
F dS = div(F) d V = 0
S W

17. Let W be the region in Figure 16 bounded by the cylinder x 2 + y 2 = 9, the plane z = x + 1, and the x y-plane. Use
the Divergence Theorem to compute the flux of F = z, x, y + 2z through the boundary of W.

y
FIGURE 16

SOLUTION We compute the divergence of F = z, x, y + 2z:


div(F) = (z) + (x) + (y + 2z) = 2
x y z
By the Divergence Theorem we have
  
F dS = div(F) d V = 2 dV
S W W
To compute the triple integral, we identify the projection D of the region on the x y-plane. D is the region in the x y plane
enclosed by the circle x 2 + y 2 = 9 and the line 0 = x + 1 or x = 1. We obtain the following integral:
    x+1  x+1 

F dS = 2 dV = 2 dz d x d y = 2z  dx dy = (2x + 2) d x d y
S W D 0 D z=0 D

We compute the double integral as the difference of two integrals: the integral over the disk D2 of radius 3, and the
integral over the part D1 of the disk, shown in the figure.
1256 C H A P T E R 18 F U N D A M E N TA L TH E O R E M S O F V E C T O R A N A LY S I S (ET CHAPTER 17)

D1
x
3

We obtain
  
F dS = (2x + 2) d x d y (2x + 2) d x d y
S D2 D1
  
= 2x d x d y + 2 dx dy (2x + 2) d x d y
D2 D2 D1

D2

x
3

The first integral is zero due to symmetry. The second integral is twice the area of D2 , that is, 2 32 = 18 . Therefore,
 
F dS = 18 (2x + 2) d x d y
S D1

We compute the double integral over the upper part of D1 . Due to symmetry, this integral is equal to half of the integral
over D1 .
y

q
x
1 3

We describe the region in polar coordinates:


1
1.91 , r 3
cos
Then
   3
(2x + 2) d x d y = 2 (2r cos + 2)r dr d
D1 1.91 1/ cos
  3  3
4r 3 cos 
= (4r 2 cos + 4r ) dr d = + 2r 2  d
1.91 1/ cos 1.91 3 1
r = cos
  
4 cos 2
= 36 cos + 18 + d
1.91 3 cos3 cos2
   
2 1 2 
= 36 cos + 18 d = 36 sin + 18 tan 
3 cos 2 3 
1.91 1.91
S E C T I O N 18.3 Divergence Theorem (ET Section 17.3) 1257
 
2
= 18 36 sin 1.91 + 18 1.91 tan 1.91 = 13.67
3
so we have
 
F dS = 18 (2x + 2) d x d y = 70.22.
S D1

18. Find a constant c for which the velocity field

F = (cx y)i + (y z)j + (3x + 4cz)k

of a fluid is incompressible [meaning that div(F) = 0].


SOLUTION We compute the divergence of F:


div(F) = (cx y) + (y z) + (3x + 4cz) = c + 1 + 4c = 5c + 1
x y z

Therefore, div(F) = 0 if 5c + 1 = 0 or c = 15 .
19. Volume as a Surface Integral Let F = x, y, z. Prove that if W is a region R3 with a smooth boundary S, then

1
Volume(W) = F dS 9
3 S

SOLUTION Using the volume as a triple integral we have



Volume(W) = 1 dV (1)
W

We compute the surface integral of F over S, using the Divergence Theorem. Since div(F) = x (x) + y (y) + z
(z) =

3, we get
   
F dS = div(F) d V = 3 dV = 3 1 dV (2)
S W W W
We combine (1) and (2) to obtain

F dS = 3 volume(W)
S
or

1
Volume(W) = F dS
3 S

20. Use Eq. (9) to calculate the volume of the unit ball as a surface integral over the unit sphere.
SOLUTION Let S be the unit sphere and W is the unit ball. By Eq. (9) we have

1
Volume(W) = F dS, F = x, y, z
3 S
To compute the surface integral, we parametrize S by

( , ) = (cos sin , sin sin , cos ), 0 2 , 0




n = sin cos sin , sin sin , cos

Then
 
 
F ( , ) n = cos sin , sin sin , cos cos sin2 , sin sin2 , cos sin

= cos2 sin3 + sin2 sin3 + cos2 sin = sin3 (cos2 + sin2 ) + cos2 sin

= sin3 + cos2 sin = sin3 + (1 sin2 ) sin = sin

We obtain the following integral:


     
1 2 1 2  2 4
Volume(W) = sin d d = 2 sin d = cos  = (1 + 1) =
3 0 0 3 0 3 0 3 3
1258 C H A P T E R 18 F U N D A M E N TA L TH E O R E M S O F V E C T O R A N A LY S I S (ET CHAPTER 17)

21. Show that (a cos sin , b sin sin , c cos ) is a parametrization of the ellipsoid
x 2 y 2 z 2
+ + =1
a b c
Then use Eq. (9) to calculate the volume of the ellipsoid as a surface integral over its boundary.
SOLUTION For the given parametrization,

x = a cos sin , y = b sin sin , z = c cos (1)

We show that it satisfies the equation of the ellipsoid


x 2 y 2 z 2  a cos sin 2  b sin sin 2  c cos 2
+ + = + +
a b c a b c

= cos2 sin2 + sin2 sin2 + cos2

= sin2 (cos2 + sin2 ) + cos2

= sin2 + cos2 = 1

Conversely, for each (x, y, z) on the ellipsoid, there exists and so that (1) holds. Therefore ( , ) parametrizes the
whole ellipsoid. Let W be the interior of the ellipsoid S. Then by Eq. (9):

1
Volume(W) = F dS, F = x, y, z
3 S
We compute the surface integral, using the given parametrization. We first compute the normal vector:


= a sin sin , b cos sin , 0



= a cos cos , b sin cos , c sin

 
= ab sin2 sin cos k ac sin sin2 j ab cos2 sin cos k bc cos sin2 i

 
= bc cos sin2 , ac sin sin2 , ab sin cos

Hence, the outward pointing normal is


 
n = bc cos sin2 , ac sin sin2 , ab sin cos
 
 
F ( , ) n = a cos sin , b sin sin , c cos bc cos sin2 , ac sin sin2 , ab sin cos

= abc cos2 sin3 + abc sin2 sin3 + abc sin cos2


= abc sin3 (cos2 + sin2 ) + abc sin cos2

= abc sin3 + abc sin cos2 = abc sin3 + abc sin (1 sin2 )
= abc sin

We obtain the following integral:


  
1 1 2
Volume(W) = F dS = abc sin d d
3 S 3 0 0
   
2 abc 2 abc  4 abc
= sin d = cos  =
3 0 3 0 3

22. Prove the identity

div(curl(F)) = 0

SOLUTION Let F = P(x, y, z), Q(x, y, z), R(x, y, z). We compute the curl of F:
 
 i j k 
  
  R Q P R Q P

curlF =  = , ,
 x y z  y z z x x y
 
 P Q R 
S E C T I O N 18.3 Divergence Theorem (ET Section 17.3) 1259

The divergence of curl(F) is thus


     
R Q P R Q P
div(curlF) = + +
x y z y z x z x y
2 R 2 Q 2 P 2 R 2 Q 2 P
= + +
xy xz yz y x z x zy
     
2 R 2 R 2 Q 2 Q 2 P 2 P
= + +
xy y x z x xz yz zy

Since the second-order partials are continuous, the mixed partials are equal. Therefore,

div (curl(F)) = 0

23. Find and prove a Product Rule expressing div( f F) in terms of div(F) and f .
SOLUTION Let F = P, Q, R. We compute div( f F):

div( f F) = div  f P, f Q, f R = ( f P) + ( f Q) + ( f R)
x y z
Applying the product rule for scalar functions we obtain
     
P f Q f R f
div( f F) = f + P + f + Q + f + R
x x y y z z
 
P Q R f f f
= f + + + P+ Q+ R = f div(F) + F f
x y z x y z
We thus proved the following identity:

div( f F) = f div(F) + F f

24. Prove the identity

div(F G) = curl(F) G F curl(G)

Then prove that the cross product of two irrotational vector fields is incompressible [F is called irrotational if curl(F) = 0
and incompressible if div(F) = 0].
SOLUTION We compute the left-hand side of the identity. For F = P, Q, R and G = S, T, U  we have
 
 i j k 

F G =  P Q R  = (QU RT )i (PU RS)j + (P T Q S)k
 S T U 


div(F G) = (QU RT ) (PU RS) + (P T Q S)
x y z
 
= (Q x U + QUx Rx T RTx ) Py U + PU y R y S RS y + (Pz T + P Tz Q z S Q Sz )
= S(R y Q z ) + T (Pz Rx ) + U (Q x Py ) P(U y Tz ) Q(Sz Ux ) R(Tx S y )

We compute the right hand side of the given identity. We have




curl(F) = R y Q z , Pz Rx , Q x Py


curl(G) = U y Tz , Sz Ux , Tx S y

Thus,


G curl(F) F curl(G) = S, T, U  R y Q z , Pz Rx , Q x Py


P, Q, R U y Tz , Sz Ux , Tx S y
= S(R y Q z ) + T (Pz Rx ) + U (Q x Py )
P(U y Tz ) Q(Sz Ux ) R(Tx S y )

This, with (1), proves the identity

div(F G) = G curl(F) F curl(G).

Thus, if both F and G are irrotational (that is, with curl zero), then their cross product is source-free (that is, with
divergence zero), as div F G = G 0 F 0 = 0.
1260 C H A P T E R 18 F U N D A M E N TA L TH E O R E M S O F V E C T O R A N A LY S I S (ET CHAPTER 17)

25. Prove that div( f g) = 0.


SOLUTION We compute the cross product:
 
 i j k 


 
f g = f x , f y , f z gx , g y , gz =  f x f y f z 

 g x g y gz 


= f y gz f z g y , f z g x f x gz , f x g y f y g x

We now compute the divergence of this vector. Using the Product Rule for scalar functions and the equality of the mixed
partials, we obtain

div( f g) = ( f y gz f z g y ) + ( f z g x f x gz ) + ( f x g y f y g x )
x y z
= f yx gz + f y gzx f zx g y f z g yx + f zy gx + f z gx y f x y gz f x gzy + f x z g y + f x g yz
f yz gx f y gx z
= ( f yx f x y )gz + (gzx gx z ) f y + ( f x z f zx )g y + (gx y g yx ) f z
+ ( f zy f yz )gx + (g yz gzy ) f x = 0

In Exercises 2628, let  denote the Laplace operator defined by

2 2 2
 = + 2 + 2
x 2 y z

26. Prove the identity

curl(curl(F)) = (div(F)) F

where F denotes F1 , F2 , F3 .


SOLUTION We compute the left-hand side of the identity. We have
 
 i j k 
  
  F3 F2 F1 F3 F2 F1

curl(F) =  = , ,
 x y z  y z z x x y
 
 F1 F2 F3 

Hence,
        
F2 F1 F1 F3 F3 F2 F2 F1
curl (curl(F)) = , ,
y x y z z x z y z x x y
   
F1 F3 F3 F2

x z x y y z

2 F2 2 F1 2 F1 2 F3 2 F3 2 F2 2 F2 2 F1
= + , + ,
y x y 2 z 2 z x zy z 2 x 2 xy

2 F1 2 F3 2 F3 2 F2
+ (1)
xz x2 y 2 yz

We now compute the right-hand side of the given identity:


 
F1 F2 F3
(div(F)) = + +
x y z
      
F1 F2 F3 F1 F2 F3 F1 F2 F3
= + + , + + , + +
x x y z y x y z z x y z
 
2 F1 2 F2 2 F3 2 F1 2 F2 2 F3 2 F1 2 F2 2 F3
= + + , + + , + +
x 2 xy xz y x y 2 yz z x zy z 2

Therefore,
 
2 F1 2 F1 2 F1 2 F2 2 F2 2 F2 2 F3 2 F3 2 F3
(div(F)) F = (div(F)) + + , + + , + +
x2 y 2 z 2 x 2 y 2 z 2 x 2 y 2 z 2
S E C T I O N 18.3 Divergence Theorem (ET Section 17.3) 1261

2 F2 2 F3 2 F1 2 F1 2 F1 2 F3 2 F2 2 F2
= + , + ,
xy xz y 2 z 2 y x yz x 2 z 2

2 F1 2 F2 2 F3 2 F3
+ (2)
z x zy x2 y 2

Since the mixed partials are equal, the expressions obtained in (1) and (2) are the same. This proves the desired identity.
27. A function satisfying  = 0 is called harmonic.
(a) Show that  = div( ) for any function .
(b) Show that is harmonic if and only if div( ) = 0.
(c) Show that if F is the gradient of a harmonic function, then curl(F) = 0 and div(F) = 0.
 
1
(d) Show F = x z, yz, (x 2 y 2 ) is the gradient of a harmonic function. What is the flux of F through a closed
2
surface?
SOLUTION
(a) We compute the divergence of :
 
2 2 2
div( ) = div , , = + 2 + 2 = 
x y z x 2 y z
(b) In part (a) we showed that  = div( ). Therefore  = 0 if and only if div( ) = 0. That is, is harmonic if
and only if is divergence free.
(c) We are given that F = , where  = 0. In part (b) we showed that

div(F) = div( ) = 0

We now show that curl(F) = 0. We have


 
 i j k 


 
curl(F) = curl( ) = curl x , y , z =  
 x y z 
 
 x y z 


= zy yz , x z zx , yx x y = 0, 0, 0 = 0

The last equality is due to the equality of the mixed partials.


 
x 2 y 2 2 2
(d) We first show that F = x z, yz, 2 is the gradient of a harmonic function. We let = x2 z y2 z such that
F = . Indeed,
   
x 2 y2
= , , = x z, yz, =F
x y z 2

We show that is harmonic, that is,  = 0. We compute the partial derivatives:

2
= xz =z
x x2
2
= yz = z
y y 2
x 2 y2 2
= =0
z 2 z 2
Therefore,

2 2 2
 = + 2 + 2 = zz+0=0
x 2 y z
Since F is the gradient of a harmonic function, we know by part (c) that div(F) = 0. Therefore, by the Divergence
Theorem, the flux of F through a closed surface is zero:
  
F dS = div(F) d V = 0 dV = 0
S W W

28. Let F = n er , where n is any number, = (x 2 + y 2 + z 2 )1/2 , and er = 1 x, y, z is the unit radial vector.
1262 C H A P T E R 18 F U N D A M E N TA L TH E O R E M S O F V E C T O R A N A LY S I S (ET CHAPTER 17)

(a) Calculate div(F).


(b) Use the Divergence Theorem to calculate the flux of F through the surface of a sphere of radius R centered at the
origin. For which values of n is this flux independent of R?
(c) Prove that (n ) = n n1 er .
(d) Use (c) to show that F is a gradient vector field for n  = 1. Then show that F = 1 er is also a gradient vector
field by computing the gradient of ln .

(e) What is the value of F ds, where C is a closed curve?
C
(f) Find the values of n for which the function = n is harmonic.
SOLUTION
(a) F is the vector field:
(n1)/2
F(x, y, z) = n 1 x, y, z = (x 2 + y 2 + z 2 ) x, y, z (1)

Hence,
F1 2 (n1)/2
n 1 (n3)/2 (n1)/2
= (x + y 2 + z 2 ) x = (x 2 + y 2 + z 2 ) 2x x + (x 2 + y 2 + z 2 )
x x 2
(n3)/2
(n3)/2
= (x 2 + y 2 + z 2 ) (n 1)x 2 + x 2 + y 2 + z 2 = (x 2 + y 2 + z 2 ) (nx 2 + y 2 + z 2 )

Similarly,
F2 2 (n1)/2
(n3)/2 2
= (x + y 2 + z 2 ) y = (x 2 + y 2 + z 2 ) (x + ny 2 + z 2 )
y y
and
F3 2 (n1)/2
(n3)/2 2
= (x + y 2 + z 2 ) z = (x 2 + y 2 + z 2 ) (x + y 2 + nz 2 )
z z
The divergence of F is the sum
(n3)/2
div(F) = (x 2 + y 2 + z 2 ) = (nx 2 + y 2 + z 2 + x 2 + ny 2 + z 2 + x 2 + y 2 + nz 2 )
(n3)/2 (n1)/2
= (x 2 + y 2 + z 2 ) (n + 2)(x 2 + y 2 + z 2 ) = (n + 2)(x 2 + y 2 + z 2 ) = (n + 2)n1

(b) Let W denote the ball inside the sphere of radius R. We may apply the Divergence Theorem when the components
of F in (1) are defined and have continuous derivatives in W, that is, when n1 n3
2 0 and 2 0 or when n 3. In this
case, we have
  
F dS = div(F) d V = (n + 2)n1 d V
S W W
We compute the triple integral by converting it to spherical coordinates. We obtain
  2   R   R
F dS = (n + 2)n1 2 sin d d d = 2 (n + 2) n+1 sin d d
S 0 0 0 0 0
      
R  R
= 2 (n + 2) n+1 d sin d = 2 (n + 2) cos  n+1 d
0 0 =0 0
 R 
n+2  R R n+2
= 4 (n + 2) n+1 d = 4 (n + 2)  = 4 (n + 2) = 4 R n+2
0 n + 2 =0 n+2

That is,

F dS = 4 R n+2 , n 3.
S
We now consider the case n < 3. We evaluate the surface integral directly, using the parametrization, ( , ) =
(R cos sin , R sin sin , R cos ), 0 2 , 0 .

n = R 2 sin er

Then,
 
F ( , ) n = n er R 2 sin er = n R 2 sin
S E C T I O N 18.3 Divergence Theorem (ET Section 17.3) 1263

n/2
= R 2 sin R 2 cos2 sin2 + R 2 sin2 sin2 + R 2 cos2 = R n+2 sin

Hence,
  2    

F dS = R n+2 sin d d = 2 R n+2 cos  = 4 R n+2
S 0 0 0

We conclude that, for all values of n,



F dS = 4 R n+2
S
The flux is independent of R when n + 2 = 0 or n = 2.
(c) We compute the gradient of n . We first compute the partial derivatives,
2 1/2 1 1/2 x x
= (x + y 2 + z 2 ) = (x 2 + y 2 + z 2 ) 2x = = (2)
x x 2 (x + y + z )
2 2 2 1/2

Similarly,
y z
= and =
y z
Therefore,
n x
( ) = n n1 = n n1 = n n2 x
x x
n y
( ) = n n1 = n n1 = n n2 y
y y
n z
( ) = n n1 = n n1 = n n2 z
z z

The gradient of n is thus


 
(n ) = n n2 x, n n2 y, n n2 z = n n2 x, y, z = n n1 1 x, y, z = n n1 er

(d) Replacing n by n + 1 in the equality of part (c), we have

(n+1 ) = (n + 1)n er = (n + 1)F

Therefore, if n  = 1, then
 
n+1
F=
n+1

We now show that F = 1 er is also a gradient vector field. We compute the gradient of ln . Using the Chain Rule and
the partial derivatives (1), we have
1 1x x
(ln ) = = = 2
x x
1 1y y
(ln ) = = = 2
y y
1 1z z
(ln ) = = = 2
z z
Therefore, the gradient of ln is
 
x y z
(ln ) = , , = 1 1 x, y, z = 1 er = F.
2 2 2
We conclude that F is a gradient vector for all values of n.
(e) Since F is a gradient vector field, it is conservative. Therefore, the line integral of F over a closed curve is zero.
(f) Using Exercise 27 part (b), = n is harmonic if and only if
 
div (n ) = 0
1264 C H A P T E R 18 F U N D A M E N TA L TH E O R E M S O F V E C T O R A N A LY S I S (ET CHAPTER 17)

That is, by part (c),



div(n n1 er ) = ndiv n1 er = 0

or

div(n1 er ) = 0 or n=0

Using part (a) for n replaced by n 1, we have

div(n1 er ) = (n + 1)n2 = 0 n = 1 or n = 0.

We conclude that = n is harmonic for n = 1 or n = 0.


 
z
29. The electric field due to a unit electric dipole oriented in the k direction is E = , where = (x 2 + y 2 +
3
z 2 )1/2 (Figure 17). Let er = 1 x, y, z.
(a) Show that E = 3 k 3z 4 er .
(b) Calculate the flux of E through a sphere centered at the origin.
(c) Calculate div(E).
(d) Can we use the Divergence Theorem to compute the flux of E through a sphere centered at the origin?

FIGURE 17 The dipole vector field restricted to the x z-plane.

SOLUTION
(a) We first compute the partial derivatives of :
1 1/2 x
= (x 2 + y 2 + z 2 ) 2x =
x 2
1 1/2 y
= (x 2 + y 2 + z 2 ) 2y =
y 2
1 1/2 z
= (x 2 + y 2 + z 2 ) 2z = (1)
z 2

We compute the partial derivatives of z3 , using the Chain Rule and the partial derivatives in (1):

 
z 3 x 3zx
=z ( ) = z (3)4 = 3z 4 = 5 = 3z 5 x
x 3 x x
 
z 3 y
=z ( ) = z (3)4 = 3z 4 = 3z 5 y
y 3 y y
 
z z
= (z 3 ) = 1 3 + z (3)4 = 3 3z 4 = 3 3z 2 5
z 3 z z

Therefore,
 
z
E= = 3z 5 xi 3z 5 yj + (3 3z 2 5 )k
3
= 3 k 3z 4 1 (xi + yj + zk) = 3 k 3z 4 er

(b) To compute the flux S E dS we use the parametrization ( , ) = (R cos sin , R sin sin , R cos ), 0
2 , 0 :

n = R 2 sin er
S E C T I O N 18.3 Divergence Theorem (ET Section 17.3) 1265

 
We compute E ( , ) n. Since = R on S, we get
 
E ( , ) n = R 3 k 3z R 4 er R 2 sin er = R 1 sin k er 3z R 2 sin

= R 1 sin k R 1 (xi + yj + zk) 3z R 2 sin

= R 2 z sin 3z R 2 sin = 2z R 2 sin

= 2R cos R 2 sin = R 1 sin 2

Hence,
  2   
2 
E dS = R 1 sin 2 d d = sin 2 d = cos 2  =0
S 0 0 R 0 R =0

(c) We use part (a) to write the vector E componentwise:


 
E = 3 k 3z 4 er = 3 k 3z 4 1 x, y, z = 3z 5 x, 3z 5 y, 3z 2 5 + 3

To find div(E) we compute the following derivatives, using (1) and the laws of differentiation. This gives
 

(3z 5 x) = 3z (5 x) = 3z 56 x + 5 1
x x x
 
x
= 3z 56 x + 5 = 3z 7 (5x 2 2 )

Similarly,

(3z 5 y) = 3z 7 (5y 2 2 )
y
and

(3z 2 5 + 3 ) = 6z 5 3z 2 (5)6 34
z z z
z z
= 6z 5 + 15z 2 6 34 = 3z 7 (5z 2 32 )

Hence,

div(E) = 3z 7 (5x 2 2 + 5y 2 2 + 5z 2 32 ) = 15z 7 (x 2 + y 2 + z 2 2 )

= 15z 7 (2 2 ) = 0

(d) Since E is not defined at the origin, which is inside the ball W, we cannot use the Divergence Theorem to compute
the flux of E through the sphere.
30. Let E be the electric field due to a long, uniformly charged rod of radius R with charge density per unit length
(Figure 18). By symmetry, we may assume that E is everywhere perpendicular to the rod and its magnitude E(r ) depends
only on the distance r to the rod (strictly speaking, this would hold only if the rod were infinite, but it is nearly true if the

rod is long enough). Show that E(r ) = for r > R. Hint: Apply Gausss Law to a cylinder of radius R and of unit
2 0 r
length with its axis along the rod.

r
R
y
x Charged rod

FIGURE 18

SOLUTION Gausss Theorem asserts that if S is a closed surface, then the total charge Q enclosed by S is given by

Q= 0 E dS
S
1266 C H A P T E R 18 F U N D A M E N TA L TH E O R E M S O F V E C T O R A N A LY S I S (ET CHAPTER 17)

where 0 is the dielectric coefficient in vacuum. Here, S is a closed cylinder of radius r and unit length that encloses a
charge Q = 1 = . Therefore, by Gausss Theorem, for r > R,
  
= 0 E dS = 0 E dS + 0 E dS
S integral cylinder bases

E is perpendicular to the normal vectors of the two bases, hence the second integral is zero, and we have
  2  1
= 0 E dS = 0 E n d y d (1)
S 0 0

r
R
1

Since E and n point in the same direction and E = E(r ), n = r , we have

E n = En = E(r ) r

Hence, by (1),
 2  1  1
= 0 r E(r ) d y d = 2 0 r E(r ) d y = 2 0 r E(r )
0 0 0
or

E(r ) = .
2 0 r

31. Let I = F dS, where
S
 
2yz xz xy
F= , 2, 2
2

( = x 2 + y 2 + z 2 ) and S is the boundary of a region W.
(a) Check that F is divergence-free.
(b) Show that I = 0 if S is a sphere centered at the origin. Note that the Divergence Theorem cannot be used. Why not?
(c) Give an argument showing that I = 0 for all S.
SOLUTION

(a) To find div(F), we first compute the partial derivatives of = x 2 + y2 + z2:
2x x 2y y 2z z
=  = , =  = , =  =
x 2 x +y +z
2 2 2 y 2 x +y +z
2 2 2 z 2 x +y +z
2 2 2

We compute the partial derivatives:


 
2yz x
= 2yz (2 ) = 2yz (2)3 = 4yz 3 = 4x yz 4
x 2 x x
 
xz y
2 = x z (2 ) = x z (2)3 = 2zx 3 = 2x yz 4
y y y
 
xy z
2 = x y (2 ) = x y (2)3 = 2x y 3 = 2x yz 4
z z z

The divergence of F is the sum of these partials. That is,

div(F) = 4x yz 4 + 2x yz 4 + 2x yz 4 = 0

We conclude that F is divergence-free.


(b) We compute the flux of F over S, using the following parametrization:

S : ( , ) = (R cos sin , R sin sin , R cos ), 0 2 , 0

n = R 2 sin er where er = 1 x, y, z


S E C T I O N 18.3 Divergence Theorem (ET Section 17.3) 1267

We compute the dot product:


 
2yz xz xy
Fn= , 2 , 2 x, y, z 1 R 2 sin = (2x yz x yz x yz)3 R 2 sin = 0
2
 
Therefore, F ( , ) n = 0, so we have
  2   2 
 
F dS = F ( , ) nd d = 0 d d = 0
S 0 0 0 0

The Divergence Theorem cannot be used since F is not defined at the origin, which is inside the ball with the boundary
S.
(c) In part (b) we showed that F n = 0 for all values of R. Therefore, I = 0 over all spheres centered at the origin.

Further Insights and Challenges


32. Let S be the boundary surface of a region W in R3 and let Den denote the directional derivative of , where en is
the outward unit normal vector. Let  be the Laplace operator defined earlier.
(a) Use the Divergence Theorem to prove
 
Den d S =  d V
S W
(b) Show that if is a harmonic function (defined in Exercise 27), then

Den d S = 0
S

SOLUTION
(a) By the theorem on evaluating directional derivatives, Den = en , hence,
 
Den d S = en d S (1)
S S
By the definition of the vector surface integral, we have
 
dS = ( en ) d S
S S
Combining with (1) gives
 
Den d S = dS
S S
We now apply the Divergence Theorem and the identity div( ) =  shown in part (a) of Exercise 27, to write
   
Den d S = dS = div( ) d V =  d V
S S W W
(b) If is harmonic, then  = 0; therefore, by the equality of part (a) we have
  
Den d S =  d V = 0 d V = 0.
S W W

33. Assume that is harmonic. Show that div( ) =  2 and conclude that
 
Den d S =  2 d V
S W

SOLUTION In Exercise 23 we proved the following Product Rule:

div( f F) = f F + f div (F)

We use this rule for f = and F = to obtain

div ( ) = + div ( ) =  2 + div ( ) (1)

By Exercise 27 part (a),

div( ) =  (2)
1268 C H A P T E R 18 F U N D A M E N TA L TH E O R E M S O F V E C T O R A N A LY S I S (ET CHAPTER 17)

Also, since is harmonic,

 = 0 (3)

Combining (1), (2), and (3), we obtain

div( ) =  2 + 0 =  2 (4)

Now, by the Theorem on evaluating directional derivatives,

Den = en

Hence,
 
Den d S = ( en ) d S (5)
S S
By the definition of the vector surface integral we have
 
dS = ( en ) d S (6)
S S
Combining (5) and (6) and using the Divergence Theorem and equality (4), we get
   
Den d S = dS = div( ) d V =  2 d V
S S W W

34. Let F = P, Q, R be a vector field defined on R3 such that div(F) = 0. Use the following steps to show that F has
a vector potential.
(a) Let A =  f, 0, g. Show that
 
g f g f
curl(A) = , ,
y z x y
(b) Fix any value y0 and show that if we define
 y
f (x, y, z) = R(x, t, z) dt + (x, z)
y0
 y
g(x, y, z) = P(x, t, z) dt + (x, z)
y0

where and are any functions of x and z, then g/y = P and f /y = R.


(c) It remains for us to show that and can be chosen so Q = f /z g/ x. Verify that the following choice works
(for any choice of z 0 ):
 z
(x, z) = Q(x, y0 , t) dt, (x, z) = 0
z0

Hint: You will need to use the relation div(F) = 0.


SOLUTION
(a) If A =  f, 0, g, then the curl of A is the following vector field:
 
 i j k 
        
  g g f f g f g f

curl(A) =  = 0 i j+ 0 k= , ,
 x y z  y x z y y z x y
 
 f 0 g 

(b) Using the Fundamental Theorem of Calculus, we have


 y
g
(x, y, z) = P(x, t, z) dt + (x, z) = P(x, y, z) + 0 = P(x, y, z)
y y y0 y
 y
f
(x, y, z) = R(x, t, z) dt + (x, z) = R(x, y, z) + 0 = R(x, y, z)
y y y0 y
S E C T I O N 18.3 Divergence Theorem (ET Section 17.3) 1269

(c) We verify that the functions


 z
(x, z) = Q(x, y0 , t) dt, (x, z) = 0
z0

satisfy the equality


f g
Q=
z x
We differentiate to obtain
 y  y
f g
= Rz (x, t, z) dt + z (x, z) Px (x, t, z) dz x (x, z)
z x y0 y0
 y
= (Px (x, t, z) + Rz (x, t, z)) dt + z (x, z) (1)
y0

By the Fundamental Theorem of Calculus,


 z

z (x, z) = Q(x, y0 , t) dt = Q(x, y0 , z) (2)
z z 0

Also, since div(F) = 0, we have

div(F) = Px + Q y + Rz = 0 Px + Rz = Q y (3)

Substituting (2) and (3) in (1) gives


 y
f g
= Q y (x, t, z) dt + Q(x, y0 , z) = Q(x, y, z) Q(x, y0 , z) + Q(x, y0 , z) = Q(x, y, z)
z x y0

Parts (a)(c) prove that F = curl(A), or A is a vector potential for F.




35. Show that F = 2y 1, 3z 2 , 2x y has a vector potential and find one.
SOLUTION Since div(F) = x (2y 1) + y (3z 2 ) + z
(2x y) = 0, we know by Exercise 34 that F has a vector

potential A, which is

A =  f, 0, g (1)
 y  z
f (x, y, z) = R(x, t, z) dt + Q(x, y0 , t) dt
y0 z0
 y
g(x, y, z) = P(x, t, z) dt
y0

Hence, P(x, y, z) = 2y 1, Q(x, y, z) = 3z 2 , and R(x, y, z) = 2x y. We choose z 0 = y0 = 0 and find f and g:


 y  z y z
 
f (x, y, z) = 2xt dt + 3t 2 dt = xt 2  + t 3  = x y 2 + z 3
0 0 t=0 t=0
 y y

g(x, y, z) = (2t 1) dt = t t 
2 = y2 y
0 t=0
Substituting in (1) we obtain the vector potential
 
A = z 3 x y 2 , 0, y 2 y



36. Show that F = 2ye z x y, y, yz z has a vector potential and find one.
SOLUTION As shown in Exercise 34, if F is divergence free, then F has a vector potential. We show that div(F) = 0:


div(F) = (2ye z x y) + (y) + (yz z) = y + 1 + y 1 = 0
x y z
We find a vector potential A, using the result in Exercise 34:

A =  f, 0, g (1)

Using z 0 = 0, we have
 y  z
f (x, y, z) = R(x, t, z) dt + Q(x, y0 , t) dt
y0 0
1270 C H A P T E R 18 F U N D A M E N TA L TH E O R E M S O F V E C T O R A N A LY S I S (ET CHAPTER 17)
 y
g(x, y, z) = P(x, t, z) dt
y0

Hence, P(x, y, z) = 2ye z x y, Q(x, y, z) = y, and R(x, y, z) = yz z. We choose y0 = 0 and compute the functions
f and g:
 y  z    
t2z y y 2z y 2
f (x, y, z) = (t z z) dt + 0 dt = zt  = zy =z y
0 0 2 t=0 2 2
 y 
xt 2  y x y2 x
g(x, y, z) = (2te z xt) dt = t 2 e z  = y 2 ez = y 2 ez
0 2 t=0 2 2

Substituting in (1) we obtain


   
y2 x
A= z y , 0, y 2 e z
2 2

37. A vector field with a vector potential has zero flux through every closed surface in its domain. In the text, we observed
er
that although the inverse-square radial vector field F = 2 satisfies div(F) = 0, F cannot have a vector potential on its

domain {(x, y, z)  = (0, 0, 0)} because the flux of F through a sphere containing the origin is nonzero.
(a) Show that the method of Exercise 34 produces a vector potential A such that F = curl(A) on the restricted domain
D consisting of R3 with the y-axis removed.
(b) Show that F also has a vector potential on the domains obtained by removing either the x-axis or the z-axis from R3 .
(c) Does the existence of a vector potential on these restricted domains contradict the fact that the flux of F through a
sphere containing the origin is nonzero?
SOLUTION
<x,y,z>
(a) We have F(x, y, z) = er2 = 3/2 , hence
(x 2 +y 2 +z 2 )
x
P(x, y, z) = 3/2
(x 2 + y 2 + z 2 )
y
Q(x, y, z) = 3/2
(x 2 + y 2 + z 2 )
z
R(x, y, z) = 3/2
(x 2 + y 2 + z 2 )
In Exercise 34, we defined the functions (taking y0 = z 0 = 0)
 y  z  y
z z
f (x, y, z) = dt + Q(x, 0, t) dt = dt
0 (x 2 + t 2 + z 2 )3/2 0 0 (x 2 + t 2 + z 2 )3/2
 y
x
g(x, y, z) = dt
0 (x + t + z 2 )3/2
2 2

These functions are defined for (x, z)  = (0, 0), since the points with x = 0 and z = 0 are on the y-axis. (Notice that for
any fixed (x, z)  = (0, 0) the interval of integration do not intersect the y-axis, therefore they are contained in the domain
D.) For (x, z)  = (0, 0) we have by the Fundamental Theorem of Calculus
 y
g x x
= dt = = P(x, y, z)
y y 0 (x 2 + t 2 + z 2 )3/2 (x 2 + y 2 + z 2 )
3/2

 y 2 3/2 1/2
f g (x + t 2 + z 2 ) z 32 (x 2 + t 2 + z 2 ) 2z
= dt
z x 0 (x 2 + t 2 + z 2 )
3

 y 2 3/2 1/2
(x + t 2 + z 2 ) x 32 (x 2 + t 2 + z 2 ) 2x
3
dt
0 (x 2 + t 2 + z 2 )
 y 2 1/2
(x + t 2 + z 2 ) (x 2 + t 2 + z 2 3z 2 )
= 3
dt
0 (x 2 + t 2 + z 2 )
 y 2 1/2
(x + t 2 + z 2 ) (x 2 + t 2 + z 2 3x 2 )
3
dt
0 (x 2 + t 2 + z 2 )
Chapter Review Exercises 1271

 y 2
x + t 2 2z 2 + t 2 + z 2 2x 2
= 5/2
dt
0 (x 2 + t 2 + z 2 )
 y
x 2 2t 2 + z 2 y
= 5/2
dt = 3/2
= Q(x, y, z)
0 (x 2 + t 2 + z 2 ) (x 2 + y 2 + z 2 )
The last integral can be verified by showing that
 
y x 2 2y 2 + z 2
=
y (x 2 + y 2 + z 2 )3/2 (x 2 + y 2 + z 2 )
5/2

and
 y
f z z
= dt = = R(x, y, z)
y y 0 (x 2 + t 2 + z 2 )3/2 (x + y + z 2 )
2 2 3/2

We conclude that the vector A =  f, 0, g is a vector potential of F in D, since


 
g f g f
curl(A) = , , = P, Q, R = F.
y z x y
(b) Suppose we remove the x-axis. In this case, we let

A = 0, f, g
 x  y
g(x, y, z) = Q(t, y, z)dt + P(x 0 , t, z) dt
x0 y0
 x
f (x, y, z) = R(t, y, z) dt
x0

Using similar procedure to that in Exercise 34, one can show that

F = curl(A).

In removing the z-axis the proof is similar, with corresponding modifications of the functions in Exercise 34.
(c) The ball inside any sphere containing the origin must intersect the x, y, and z axes; therefore, F does not have a
vector potential in the ball, and the flux of F through the sphere may differ from zero, as in our example.

CHAPTER REVIEW EXERCISES





1. Let F(x, y) = x + y 2 , x 2 y and let C be the unit circle, oriented counterclockwise. Evaluate F ds directly as
C
a line integral and using Greens Theorem.
SOLUTION We parametrize the unit circle by c(t) = (cos t, sin t), 0 t 2 . Then, c (t) =  sin t, cos t and
F(c(t)) = (cos t + sin2 t, cos2 t sin t). We compute the dot product:
 
F(c(t)) c (t) = cos t + sin2 t, cos2 t sin t  sin t, cos t

= ( sin t)(cos t + sin2 t) + cos t (cos2 t sin t)


= cos3 t sin3 t 2 sin t cos t

The line integral is thus


  2

F (c(t)) c (t) dt = cos3 t sin3 t 2 sin t cos t dt
C 0
 2  2  2
= cos3 t dt sin3 t dt sin 2t dt
0 0 0
   
cos2 t sin t 2 sin t 2 sin2 t cos t 2 cos t 2 cos 2t 2
= + + +  + 2  =0
3 3 0 3 3 0 0

We now compute the integral using Greens Theorem. We compute the curl of F. Since P = x + y 2 and Q = x 2 y,
we have
Q P
= 2x 2y
x y
1272 C H A P T E R 18 F U N D A M E N TA L TH E O R E M S O F V E C T O R A N A LY S I S (ET CHAPTER 17)

Thus,
 
F ds = (2x 2y) d x d y
C D
y

D
x
1

We compute the double integral by converting to polar coordinates. We get


  2  1  2  1
F ds = (2r cos 2r sin )r dr d = 2r 2 (cos sin ) dr d
C 0 0 0 0
        2 
1 2 2 3 1  2
= 2r 2 dr (cos sin ) d = r sin + cos  = (1 1) = 0
0 0 3 0 0 3

2. Let R be the boundary of the rectangle in Figure 1 and let R1 and R2 be the boundaries of the two triangles, all
oriented counterclockwise.
  
(a) Determine F ds if F ds = 4 and F ds = 2.
R1  R R2

(b) What is the value of F ds if R is oriented clockwise?


R

y
Rectangle R

R1

R2

x
FIGURE 1

SOLUTION
(a) Since all boundaries are oriented counterclockwise, the segment D B is oriented in opposite directions as part of the
boundaries R1 and R2 .
y
R
D C
R1

R2
A B
x

Therefore, the contributions of this segment to the sum of the line integrals over R1 and R2 cancel each other and the
following equality holds:
  
F ds = F ds + F ds
R R1 R2

Substituting the given information, we get


 
4= F ds 2 or F ds = 6.
R1 R1

(b) Reversing the orientation of the curve gives the opposite integral. Therefore if R is oriented clockwise, the line
integral is the opposite of 4; that is,

F ds = 4.
R
Chapter Review Exercises 1273

In Exercises 36, use Greens Theorem to evaluate the line integral around the given closed curve.

3. x y 3 d x + x 3 y d y, where C is the rectangle 1 x 2, 2 y 3, oriented counterclockwise.
C
SOLUTION

3 C

x
1 2

Since P = x y 3 , Q = x 3 y the curl of F is


Q P
= 3x 2 y 3x y 2
x y
By Greens Theorem we obtain
   3  2
x y3 d x + x 3 y d y = (3x 2 y 3x y 2 ) d x d y = (3x 2 y 3x y 2 ) d x d y
C D 2 1
 3   3   
3x 2 y 2 2 3y 2
= x3 y d y = (8y 6y 2 ) y dy
2 2 x=1 2 2
 3     
9y 2 3y 3 9y 2 3 81 81
= + 9y d y = + = + (12 + 18) = 30
2 2 2 2 2 2 2


4. (3x + 5y cos y) d x + x sin y d y, where C is any closed curve enclosing a region with area 4, oriented counter-
C
clockwise.
SOLUTION The components of F are P = 3x + 5y cos y and Q = x sin y. Therefore the curl of F is

Q P
= sin y (5 + sin y) = 5
x y
Using Greens Theorem we obtain
    
Q P
P dx + Q dy = dA = (5) d A
C D x y D

= 5 1 d A = 5Area( A) = 5 4 = 20
D


5. y 2 d x x 2 d y, where C consists of the arcs y = x 2 and y = x, 0 x 1, oriented clockwise.
C
SOLUTION We compute the curl of F.

C
y= x

y = x2
x
0 1
1274 C H A P T E R 18 F U N D A M E N TA L TH E O R E M S O F V E C T O R A N A LY S I S (ET CHAPTER 17)

We have P = y 2 and Q = x 2 , hence


Q P
= 2x 2y
x y
We now compute the line integral using Greens Theorem. Since the curve is oriented clockwise, we consider the negative
of the double integrals. We get
   1  x
y2 d x x 2 d y = (2x 2y) d A = (2x 2y) d y d x
C D 0 x2
 1  x  1
  
= 2x y + y 2  dx = 2x x + x (2x x 2 + x 4 ) d x
0 y=x 2 0
 1 
x5 x4 4x 5/2 x 2 1
= (x 4 2x 3 + 2x 3/2 + x) d x = + +
0 5 2 5 2 0
1 1 4 1 3
= + + =
5 2 5 2 5

6. ye x d x + xe y d y, where C is the triangle with vertices (1, 0), (0, 4), and (0, 1), oriented counterclockwise.
C
SOLUTION

(0, 4)

D
(0, 1)
x
(1, 0)

The components of the vector field are P = ye x and Q = xe y , hence the flux is
Q P
= e y ex .
x y
Greens Theorem implies that
    
Q P
ye x d x + xe y d y = dA = (e y e x ) d A
C0 D x y D
 0  4x+4  0 4x+4

= (e y e x ) d y d x = e y ye x  dx
1 x+1 1 y=x+1
 0
= e4x+4 (4x + 4)e x e x+1 (x + 1)e x dx
1
 0
= e4x+4 e x+1 (3x + 3)e x d x
1
 0  0
= (e4x+4 e x+1 3e x ) d x 3xe x d x
1 1

y = 4x + 4
D
1
x
1 y=x+1

The second integral is computed by parts. We obtain


 0 0
e4x+4  
ye x d x + xe y d y = e x+1 3e x  3e x (x 1)
C 4 1 1
Chapter Review Exercises 1275
   
e4 1
= e3 1 3e1 (3 + 6e1 )
4 4

e4 3
= e + 3e1 10.58
4 4
 
7. Let c(t) = t 2 (1 t), t(t 1)2 .
(a) Plot the path c(t) for 0 t 1.
(b) Calculate the area of the region enclosed by c(t) for 0 t 1.
SOLUTION
(a) The path c(t) for 0 t 1 is shown in the figure:
y

0.1

x
0 0.1

Note that the path is traced out clockwise as t goes from 0 to 1.


(b) We use the formula for the area enclosed by a closed curve,

1
A= (x d y y d x)
2 C

We compute the line integral. Since x = t 2 (1 t) and y = t(t 1)2 , we have



d x = 2t (1 t) t 2 dt = 2t 3t 2 dt

d y = (t 1)2 + t 2(t 1) = (t 1)(3t 1) dt

Therefore,

x d y y d x = t 2 (1 t) (t 1)(3t 1) dt t(t 1)2 (2t 3t 2 ) dt = t 2 (t 1)2 dt

We obtain the following integral (note that the path must be counterclockwise):
    
1 0 2 2 1 1 4 3 2 1 t5 t4 t 3 1 1
A= t (t 1) dt = (t 2t + t ) dt = +  =
2 1 2 0 2 5 2 3 0 60

8. In (a)(d), state whether the equation is an identity (valid for all F or ). If it is not, provide an example in which the
equation does not hold.
(a) curl( ) = 0 (b) div( ) = 0
(c) div(curl(F)) = 0 (d) (div(F)) = 0
SOLUTION
(a) This equality is valid for all since


= x , y , z
 
 i j k 
 
 
 
curl( ) =   = (zy yz )i (zx x z )j ( yx x y )k
 x y z 
 
 x y z 

By the equality of the mixed partials, we conclude that curl( ) is the zero vector.
(b) This equation is not an identity. Take (x, y, z) = x 2 + y + z. Then = 2x, 1, 1 and

div( ) = (2x) + (1) + (1) = 2 + 0 + 0 = 2  = 0.
x y z
1276 C H A P T E R 18 F U N D A M E N TA L TH E O R E M S O F V E C T O R A N A LY S I S (ET CHAPTER 17)

(c) The equality div (curl(F)) = 0 is an identity. To prove it, we let F = P, Q, R. Then
 
 i j k 

 

curlF = F =   = (R y Q z )i (Rx Pz )j + (Q x Py )k
 x y z 
 
 P Q R 

div (curl(F)) = (R y Q z ) (Rx Pz ) + (Q x Py )
x y z
= R yx Q zx (Rx y Pzy ) + Q x z Pyz
= (R yx Rx y ) + (Pzy Pyz ) + (Q x z Q zx ) = 0

The last equality is due the equality of the mixed partials.


 
(d) The equality (div(F)) = 0 is not an identity. Take F = x 2 , y, z . Then

2
div(F) = (x ) + (y) + (z) = 2x + 2
x y z
 

(div(F)) = (2x + 2), (2x + 2), (2x + 2) = 2, 0, 0  = 0
x y z

In Exercises 912, calculate the curl and divergence of the vector field.

9. F = yi zk
SOLUTION We compute the curl of the vector field,
 
 i j k 

 

curl(F) =  
 x y z 
 
 y 0 z 
     
(0) (y)
= (z) (0) i (z) (y) j + k
y z x z x y
= 0i + 0j 1k = k

The divergence of F is

div(F) = (y) + (0) + (z) = 0 + 0 1 = 1.
x y z


10. F = e x+y , e y+z , x yz


SOLUTION The curl of F = e x+y , e y+z , x yz is the following vector:
 
 i j k 

 

curl(F) = F =  
 x y z 
 x+y 
 e e y+z x yz 
     
y+z x+y
= (x yz) e y+z i (x yz) e x+y j + e e k
y z x z x y


= (x z e y+z )i (yz)j e x+y k = x z e y+z , yz, e x+y

The divergence of F is
x+y y+z
divF = (e )+ (e ) + (x yz) = e x+y + e y+z + x y.
x y z

11. F = (ex y z )
2 2 2


In Exercise 8 we proved the identity curl( ) = 0. Here, = ex y z , and we have curl ex y z
2 2 2 2 2 2
SOLUTION =
0. To compute div F, we first write F explicitly:
 
F = ex y z = 2xex y z , 2yex y z , 2zex y z = P, Q, R
2 2 2 2 2 2 2 2 2 2 2 2
Chapter Review Exercises 1277

P Q R
div(F) = + +
x y z

= 2ex y z + 4x 2 ex y z + 2ex y z + 4y 2 ex y z
2 2 2 2 2 2 2 2 2 2 2 2


+ 2ex y z + 4z 2 ex y z
2 2 2 2 2 2


= 2ex y z 2(x 2 + y 2 + z 2 ) 3
2 2 2

  
12. e = 1 x, y, z = x 2 + y 2 + z 2

SOLUTION It can be easily verified that e = for (x, y, z) = x 2 + y 2 + z 2 = . Therefore, by the identity
curl( ) = 0 (provided in Exercise 8), we have

curl(e ) = curl( ) = 0

We compute the divergence of e . Since x = x , y = y , z = z , we have


div(e ) = (x 1 ) + (y 1 ) + (z 1 ) = (1 x 2 x ) + (1 y 2 y ) + (1 z 2 z )
x y z
 
x2 y2 z2 2
= 31 2 (x x + y y + z z ) = 31 2 + + = 31 2 = 21

13. Recall that if F1 , F2 , and F3 are differentiable functions of one variable, then

curl (F1 (x), F2 (y), F3 (z)) = 0

Use this to calculate the curl of



3
F = x 2 + y 2 , ln y + z 2 , z 3 sin(z 2 )e z

SOLUTION We use the linearity of the curl and the property mentioned in the exercise to compute the curl of F:
 3   3
  
curl F = curl x 2 + y 2 , ln y + z 2 , z 3 sin z 2 e z = curl x 2 , ln y, z 3 sin(z 2 )e z + curl y 2 , z 2 , 0
   2 2

= 0 + curl y 2 , z 2 , 0 = (0) z 2 , y 2 (0), z y = 2z, 0, 2y
y z z x x y

14. Give an example of a nonzero vector field F such that curl(F) = 0 and div(F) = 0.
SOLUTION Let F = x, y, 0. Then
 
 i j k 
      
 

curl(F) =  = (0) + (y) i (0) (x) j + (y) (x) k = 0
 x y z  y z x z x y
 
 x y 0 

div(F) = (x) + (y) + (0) = 1 1 + 0 = 0
x y z


15. Verify the identities of Exercises 22 and 24 in Section 18.3 for the vector fields F = x z, ye x , yz and G =

2
z , x y3, x 2 y .


SOLUTION We first show div(curl(F)) = 0. Let F = P, Q, R = x z, ye x , yz . We compute the curl of F:
 
 i j k 
  
  R Q P R Q P

curlF =  = , ,
 x y z  y z z x x y
 
 P Q R 

Substituting in the appropriate values for P, Q, R and taking derivatives, we get




curlF = z 0, x 0, ye x 0

Thus,

div (curl(F)) = (z)x + (x) y + (ye x )z = 0 + 0 + 0 = 0.


1278 C H A P T E R 18 F U N D A M E N TA L TH E O R E M S O F V E C T O R A N A LY S I S (ET CHAPTER 17)
 
Likewise, for G = P, Q, R = z 2 , x y 3 x 2 y , we compute the curl of G:
 
 i j k 
   
  R Q P R Q P
 
curlG =  = , ,
 x y z  y z z x x y
 
 P Q R 

Substituting in the appropriate values for P, Q, R and taking derivatives, we get


 
curlG = x 2 0, 2z 2x y, y 3 0

Thus,

div (curl(G)) = (x 2 )x + (2z 2x y) y + (y 3 )z = 2x 2x = 0.





We now work on the second identity. For F = x z, ye x , yz and G = z 2 , x y 3 , x 2 y , it is easy to calculate

F G = x 2 y 2 e x x y 4 z, yz 3 x 3 yz, x 2 y 3 z yz 2 e x 

Thus,

div(F G) = (2x y 2 e x + x 2 y 2 e x y 4 z) + (z 3 x 3 z) + (x 2 y 3 2yze x )

On the other hand, from our work above,




curlF = z, x, ye x
 
curlG = x 2 , 2z 2x y, y 3

So, we calculate

G curl F F curl G = z 2 z + x y 3 x + x 2 y ye x x z x 2 ye x (2z 2x y) yz y 3


= z 3 + x 2 y 3 + x 2 y 2 e x + 2x y 2 e x x 3 z 2yze x y 4 z
= (2x y 2 e x + x 2 y 2 e x y 4 z) + (z 3 x 3 z) + (x 2 y 3 2yze x ) = div(F G)

16. Suppose that S1 and S2 are surfaces with the same oriented boundary curve C. Which of the following conditions
guarantees that the flux of F through S1 is equal to the flux of F through S2 ?
(a) F = for some function
(b) F = curl(G) for some vector field G
SOLUTION If F = curl(G), then by the Theorem on Surface Independence for Curl Vector Fields, the flux of F through
a surface S depends only the oriented boundary S. Since S1 and S2 have the same oriented boundary curve, we conclude
that the flux of F through S1 is equal to the flux of F through S2 . The condition in (a) that F is conservative does not
guarantee that the flux of F through S1 is equal to the flux through S2 .
17. Prove that if F is a gradient vector field, then the flux of curl(F) through a smooth surface S (whether closed or not)
is equal to zero.
SOLUTION If F is a gradient vector field, then F is conservative; therefore the line integral of F over any closed curve
is zero. Combining with Stokes Theorem yields
 
curl(F) dS = F ds = 0
S S

18. Verify Stokes Theorem for F = y, z x, 0 and the surface z = 4 x 2 y 2 , z 0, oriented by outward-pointing
normals.

SOLUTION We begin by computing the line integral C F ds. The boundary curve is the circle x 2 + y 2 = 4 (in the
x y plane) oriented in the counterclockwise direction. We use the parametrization

C : r (t) = (2 cos t, 2 sin t, 0), 0 t 2

Then,

F (r (t)) = 2 sin t, 2 cos t, 0


r  (t) = 2 sin t, 2 cos t, 0
Chapter Review Exercises 1279

F (r (t)) r  (t) = 2 sin t, 2 cos t, 0 2 sin t, 2 cos t, 0 = 4 sin2 t 4 cos2 t = 4

The line integral is thus


  2
F ds = 4 dt = 8 (1)
C 0

We now compute the integral S curl(F) dS. We find the curl of F:
 
 i j k 
      
 

curl(F) =   = 0 (z x) i 0 y j+ (z x) y k
 x y z  z z x y
 
 y zx 0 
= i 2k = 1, 0, 2

We parametrized the surface by



(u, v) = u cos v, u sin v, 4 u 2 , 0 v < 2 , 0 u 2.

Then,

Tu = = cos v, sin v, 2u
u

Tv = = u sin v, u cos v, 0
v
 
 i j k 
 
Tu Tv =  cos v sin v 2u  = (2u 2 cos v)i + (2u 2 sin v)j + (u cos2 v + u sin2 v)k

 u sin v u cos v 0 
 
= 2u 2 cos v, 2u 2 sin v, u

The surface is oriented outwards, hence the z-component of the normal vector is nonnegative. Therefore, the normal
vector is (recall that u 0)
 
n = 2u 2 cos v, 2u 2 sin v, u

We compute the dot product:


 
curl(F) n = 1, 0, 2 2u 2 cos v, 2u 2 sin v, u = 2u 2 cos v 2u

We obtain the following integral:


  2  2  2 2
2u 3 cos v 
curl(F) n ds = (2u 2 cos v 2u) du dv = u 
2 dv
S 0 0 0 3 u=0
 2    2
16 cos v 16 sin v 
= 4 dv = 4v  = 8 (2)
0 3 3 0

By (1) and (2), both the line integral and the flux of the curl are equal to 8 . Thus, this example verifies Stokes
Theorem.

x2
In Exercises 1920, let F = z 2 , x + z, y 2 and let S be the upper half of the ellipsoid + y 2 + z 2 = 1, oriented by
4
outward-pointing normals.

19. Use Stokes Theorem to compute curl(F) dS.
S
 
SOLUTION We compute the curl of F = z 2 , x + z, y 2 :
 
 i j k 
 
 
 
curl(F) =   = (2y 1)i (0 2z)j + (1 0)k = 2y 1, 2z, 1
 x y z 
 
 z2 x +z y2 
1280 C H A P T E R 18 F U N D A M E N TA L TH E O R E M S O F V E C T O R A N A LY S I S (ET CHAPTER 17)

2
Let C denote the boundary of S, that is, the ellipse x4 + y 2 = 1 in the x y-plane, oriented counterclockwise. Then by
Stokes Theorem we have
 
curl(F) dS = F ds (1)
S C
We parametrize C by

C : r (t) = (2 cos t, sin t, 0), 0 t 2

Then
 
F (r (t)) r  (t) = 0, 2 cos t, sin2 t 2 sin t, cos t, 0 = 2 cos2 t

Combining with (1) gives


  2 
sin 2t 2
curl(F) ds = 2 cos2 t dt = t + = 2
S 0 2 0

20. Compute F dS. Hint: Find a vector potential A, that is, a vector field A such that F = curl (A), and use
S
Stokes Theorem.
SOLUTION If there exists a vector field A such that F = curl(A), then by Stokes Theorem
  
F dS = curl(A) dS = A ds (1)
S S C
We need to find a vector field A =  A1 , A2 , A3  such that
A3 A2 A1 A3 A2 A1
= z2, = x + z, = y2
y z z x x y
We let
 
y3 z2 z3 x2
A= + , ,
3 2 3 2

Substituting in (1) we obtain


   
y3 z2 z3 x2
F dS = + , , ds (2)
S C 3 2 3 2
2
We compute the local integral. The boundary C is the intersection of the ellipsoid x4 + y 2 + z 2 = 1 with the x y-plane,
2
that is, the ellipse x4 + y 2 = 1. We parametrize C by the following parametrization (oriented counterclockwise):

r (t) = (2 cos t, sin t, 0), 0 t 2

Then
 
sin3 t 4 cos2 t 2 4
A (r (t)) r  (t) = , 0, 2 sin t, cos t, 0 = sin t
3 2 3

By (2), we obtain the following flux:


  2     2

F dS =
2 4
sin t dt =
2

sin3 t cos t
+
3 t

sin 2t  =
3 3 4 4 2 4  2
S 0 0



21. Use Stokes Theorem to evaluate y, z, x ds, where C is the curve in Figure 2.
C
z

(0, 0, 1)
y2 + z2 = 1

x (0, 1, 0) y

FIGURE 2
Chapter Review Exercises 1281

SOLUTION We compute the curl of F = y, z, x:


 
 i j k 
 
 
 
curl(F) =   = i j k = 1, 1, 1
 x y z 
 
 y z x 

By Stokes Theorem, we have


  
y, z, x ds = curl(F) dS = (curl(F) en ) dS
C S S
Since the boundary C of the quarter circle S is oriented clockwise, the induced orientation on S is normal pointing in the
negative x direction. Thus,

en = 1, 0, 0 .

Hence,

curl(F) en = 1, 1, 1 1, 0, 0 = 1.

Combining with (1) we get


 

y, z, x ds = 1 ds = Area (S) =
C S 4

22. Verify the Divergence Theorem for F = 0, 0, z and the region x 2 + y 2 + z 2 = 1.
SOLUTION Let S be the boundary of the unit sphere W. We calculate both sides of the equation:
 
F dS = div (F) d V (1)
S W
We start with the surface integral. We parametrize S by


( , ) = cos sin , sin sin , cos , 0 < 2 , 0

Then (see Example 1 in Section 17.5)




n = sin cos sin , sin sin , cos

Hence,
 


F ( , ) n = 0, 0, cos sin cos sin , sin sin , cos = cos2 sin

We obtain the following integral:


  2   2 
 
F dS = F ( , ) n d d = cos2 sin d d
S 0 0 0 0
     
cos3  1+1 4
= 2 cos2 sin d = 2 = 2 = (2)
0 3 0 3 3

We now compute the triple integral in (1). We find the divergence of F:



div(F) = (0) + (0) + (z) = 1
x y z
Hence,
 
4
div(F) d V = 1 d V = Volume (W) = (3)
W W 3
The integrals in (2) and (3) are equal, as follows from the Divergence Theorem.

In Exercises 2326, use the Divergence Theorem to calculate F dS for the given vector field and surface.
S


23. F = x y, yz, x 2 z + z 2 , S is the boundary of the box [0, 1] [2, 4] [1, 5].
SOLUTION
1282 C H A P T E R 18 F U N D A M E N TA L TH E O R E M S O F V E C T O R A N A LY S I S (ET CHAPTER 17)

z
5

1 2
x 4
y
 
We compute the divergence of F = x y, yz, x 2 z + z 2 :


div(F) = xy + yz + (x 2 z + z 2 ) = y + z + x 2 + 2z = x 2 + y + 3z
x y z
The Divergence Theorem gives
    5 4 1  5 4 3 1
x 
x y, yz, x 2 z + z 2 dS = (x 2 + y + 3z) d x d y dz = + (y + 3z)x  d y dz
S 1 2 0 1 2 3 x=0
 5 4   5 4
1 1 1 
= + y + 3z d y dz = y + y 2 + 3zy  dz
1 1 3 1 3 2 y=1
 5      5 
4 16 1 1 17
= + + 12z + + 3z dz = + 9z dz
1 3 2 3 2 1 2

17z 9z 2 5 (17 5 + 9 25) (17 + 9)
= + = = 142
2 2 1 2


24. F = x y, yz, x 2 z + z 2 , S is the boundary of the unit sphere.
SOLUTION We use spherical coordinates:

x = cos sin , y = sin sin , z = cos

with

0 2 , 0 , 01

We obtain
    
x y, yz, x 2 z + z 2 dS = div(F) d V = (x 2 + y + 3z) d V
S V V
 2   1
= (2 cos2 sin2 + sin sin + 3 cos ) 2 sin d d d
0 0 0
     1 
2
= cos2 d sin3 d 4 d
0 0 0
     1 
2
+ sin d sin2 d 3 d
0 0 0
   1 
+ 6 cos sin d 3 d
0 0

4 1 4
= +0+0=
3 5 15

2
25. F = x yz + x y, 12 y 2 (1 z) + e x , e x +y , S is the boundary of the solid bounded by the cylinder x 2 + y 2 = 16
2

and the planes z = 0 and z = y 4.


SOLUTION We compute the divergence of F:
 
y2
(1 z) + e + (e x +y ) = yz + y + y(1 z) = 2y
x 2 2
div(F) = (x yz + x y) +
x y 2 z
Chapter Review Exercises 1283

Let S denote the surface of the solid W. The Divergence Theorem gives
     0
F dS = div(F) d V = 2y d V = 2y dz d x d y
S W W D y4
 0  

= 2yz  dx dy = 2y (0 (y 4)) d x d y = (8y 2y 2 ) d x d y
D z=y4 D D

We convert the integral to polar coordinates:


  2  4
F dS = (8r cos 2r 2 cos2 )r dr d
S 0 0
       
4 2 4 2
=8 r 2 dr cos d r 3 dr 2 cos2 d
0 0 0 0
    
r 4 4 sin 2 2 44
=0  +  = 2 = 128
4 0 2 0 4



26. F = sin(yz), x 2 + z 4 , x cos(x y) , S is any smooth closed surface that is the boundary of a region in R3 .
SOLUTION We compute the divergence of F:

 2
div(F) = (sin(yz)) + x + z4 + (x cos x(x y)) = 0
x y z
Let W denote the solid inside S. The Divergence Theorem gives
  
F dS = div(F) d V = 0 dV = 0
S W W

27. Find the volume of a region W if


  
1 2
x + x y + z, x + 3y y , 4z dS = 16
W 2
 
SOLUTION Let F = x + x y + z, x + 3y 12 y 2 , 4z . We compute the divergence of F:
 
1 2
div(F) = (x + x y + z) + x + 3y y + (4z) = 1 + y + 3 y + 4 = 8
x y 2 z
Using the Divergence Theorem and the given information, we obtain
   
16 = F dS = div(F) d V = 8 dV = 8 1 d V = 8 Volume (W)
S W W W
That is,

16 = 8 Volume (W)

or

Volume (W) = 2


28. Show that the circulation of F = x 2 , y 2 , z(x 2 + y 2 ) around any curve C on the surface of the cone z 2 = x 2 + y 2
is equal to zero (Figure 3).

z2 = x2 + y2

y
x
FIGURE 3
1284 C H A P T E R 18 F U N D A M E N TA L TH E O R E M S O F V E C T O R A N A LY S I S (ET CHAPTER 17)

SOLUTION Let S be the part of the cone that is inside C. Then by Stokes Theorem, the circulation of F around C is
 
F ds = curl(F) dS (1)
C S
We parametrize the cone by

(r, ) = (r cos , r sin , r )

Then,

Tr = = cos , sin , 1
r

T = = r sin , r cos , 0

 
 i j k 
 
n = T Tr =  r sin r cos 0  = r cos , r sin , r 

 cos sin 1 

We compute the curl of F and express it in terms of the parameters:


 
 i j k 
 
 
 
curl(F) =   = 2yz, 2x z, 0
 x y z 
 
 x2 y 2 z(x 2 + y 2 ) 
 
curl(F) ((r, )) = 2r sin r, 2r cos r, 0 = 2r 2 sin , 2r 2 cos , 0

The dot product is thus


   
curl(F) (r, ) n = 2r 2 sin , 2r 2 cos , 0 r cos , r sin , r 

= 2r 3 sin cos 2r 3 cos sin + 0 = 0

We see that curl(F) is tangent to the cone at all points on the cone, hence the surface integral in (1) is zero. We conclude
that the circulation of F around any curve C on the cone is zero.

In Exercises 2932, let F be a vector field whose curl and divergence at the origin are

curl(F)(0, 0, 0) = 2, 1, 4 , div(F)(0, 0, 0) = 2



29. Estimate F ds, where C is the circle of radius 0.03 in the x y-plane centered at the origin.
C
SOLUTION We use the estimation

F ds (curl(F)(0) en ) Area(R)
C
z

en

y
C
x

The unit normal vector to the disk R is en = k = 0, 0, 1. The area of the disk is

Area (R) = 0.032 = 0.0009 .

Using the given curl at the origin, we have



F ds 2, 1, 4 0, 0, 1 0.0009 = 4 0.0009 0.0113
C
Chapter Review Exercises 1285

30. Estimate F ds, where C is the boundary of the square of side 0.03 in the yz-plane centered at the origin. Does
C
the estimate depend on how the square is oriented within the yz-plane? Might the actual circulation depend on how it is
oriented?
SOLUTION We use the estimation

F ds (curl(F)(0) en ) Area (R) (1)
C

C
R

en y

If we orient C counterclockwise, then the unit normal vector is

en = i = 1, 0, 0 .

The area of the square is Area(R) = 0.032 = 0.0009, and by the given information the curl at the origin is 2, 1, 4.
Therefore (1) gives the estimation

F ds 2, 1, 4 1, 0, 0 0.0009 = 2 0.0009 = 0.0018
C
The estimate depend on curl(F), en and the area of the square. Hence, if we flip the square over (such that en points along
the negative x-axis), then we will get a different answer.
31. Suppose that F is the velocity field of a fluid and imagine placing a small paddle wheel at the origin. Find the equation
of the plane in which the paddle wheel should be placed to make it rotate as quickly as possible.
SOLUTION The paddle wheel has the maximum spin when the circulation of the velocity field F around the wheel is
maximum. The maximum circulation occurs when en , and the curl of F at the origin (i.e., the vector 2, 1, 4) point in
the same direction. Therefore, the plane in which the paddle wheel should be placed is the plane through the origin with
the normal 2, 1, 4. This plane has the equation, 2x y + 4z = 0.
32. Estimate the flux of F through the box of side 0.5 in Figure 4. Does the result depend on how the box is oriented
relative to the coordinate axes?
z

0.5

FIGURE 4

SOLUTION We use the following estimation:



F dS div F(0) Volume (W)
S

The volume of the box W is 0.53 , and we are given that div(F)(0) = 2. This gives the estimation

F dS 2 0.53 = 0.25.
S
The negative sign shows that there is a net inflow across the box. Our estimation of the flux does not depend on the
orientation of the box; rather, it depends on the magnitude of the divergence of F.
33. The velocity field of a fluid (in meters per second) is


F = x 2 + y 2 , 0, z 2
1286 C H A P T E R 18 F U N D A M E N TA L TH E O R E M S O F V E C T O R A N A LY S I S (ET CHAPTER 17)

Let W be the region between the hemisphere


" #
S = (x, y, z) : x 2 + y 2 + z 2 = 1, x, y, z 0

and the disk


" #
D = (x, y, 0) : x 2 + y 2 1

in the x y-plane.
(a) Show that no fluid flows across D. 
(b) Use (a) to show that the rate of fluid flow across S is equal to div(F) d V . Compute this triple integral using
W
spherical coordinates.
SOLUTION
(a) To show that no fluid flows across D, we show that the normal component of F at each point on D is zero. At each
point P = (x, y, 0) on the x y-plane,
   
F(P) = x 2 + y 2 , 0, 02 = x 2 + y 2 , 0, 0 .

Moreover, the unit normal vector to the x y-plane is en = (0, 0, 1). Therefore,
 
F(P) en = x 2 + y 2 , 0, 0 0, 0, 1 = 0.

Since D is contained in the x y-plane, we conclude that the normal component of F at each point on D is zero. Therefore,
no fluid flows across D.
(b) By the Divergence Theorem and the linearity of the flux we have
  
F dS + F dS = div(F) d V
S D W
Since the flux through the disk D is zero, we have
 
F dS = div(F) d V (1)
S W
To compute the triple integral, we first compute div(F):
2
div(F) = (x + y 2 ) + (0) + (z 2 ) = 2x + 2z = 2(x + z).
x y z
z

Using spherical coordinate we get


  /2  2  1
div(F) d V = 2 ( sin cos + cos )2 sin d d
W 0 0 0
 1      /2 
/2 2
=2 3 d sin2 d cos d + 2 cos sin d
0 0 0 0
  /2   
1 cos 2  /2
= 0+ sin 2 d =  = (1 1) =
2 0 2 2 0 4 2

Combining with (1) we obtain the flux:




F dS =
S 2

34. The velocity field of a fluid (in meters per second) is

F = (3y 4)i + ey(z+1) j + (x 2 + y 2 )k


Chapter Review Exercises 1287

(a) Estimate the flow rate (in cubic meters per second) through a small surface S around the origin if S encloses a region
of volume 0.01 m3 .
(b) Estimate the circulation of F about a circle in the x y-plane of radius r = 0.1 m centered at the origin (oriented
counterclockwise when viewed from above).
(c) Estimate the circulation of F about a circle in the yz-plane of radius r = 0.1 m centered at the origin (oriented
counterclockwise when viewed from the positive x-axis).
SOLUTION
(a) We use the approximation

F dS div(F)(0)Vol(W) (1)
S

Here, Vol(W) = 0.01 m3 . We compute the divergence at the origin:


y(z+1)
div(F) = (3y 4) + e + (x 2 + y 2 ) = (z + 1)ey(z+1) div(F)(0) = 1
x y z
Substituting in (1) gives the estimation

F dS 1 0.01 = 0.01 m3 /s
S
(b) We use the estimation for the circulation using the point P = (0, 0.1, 0):

F ds (curl(F)(P) en ) Area (R) (2)
C
z

en = k

R P = (0, 0.1, 0)
y

The unit normal vector to R is en = 0, 0, 1 = k and the area of the disc is 0.12 = 0.01 . We compute the curl at P:
 
 i j k 
 
 
curl(F) =   = 2y ey(z+1) (y) i (2x 0)j + (0 3)k
x y z 
 
 (3y 4) ey(z+1) x 2 + y 2 

= 2y + yey(z+1) i 2xj 3k

Hence,

curl(F)(P) = (0.2 + 0.1e0.1 )i 3k



Dcurl(F)(P) en = (0.2 + 0.1e0.1 )i 3k k = 3

Combining with (2) gives the estimation



F ds 3 0.01 = 0.03
C
(c) We use estimation (2), only that now the unit normal vector is en = i.
z

C
R

P = (0, 0.1, 0)
y

x en = i
1288 C H A P T E R 18 F U N D A M E N TA L TH E O R E M S O F V E C T O R A N A LY S I S (ET CHAPTER 17)

We get

F ds (0.2 + 0.1e0.1 )i 3k i 0.01 = (0.2 + 0.1e0.1 ) 0.01 = 0.009
C

x
35. Let (x, y) = x + 2 . The vector field F = (Figure 5) provides a model in the plane of the velocity field
x + y2
of an incompressible, irrotational fluid flowing past a cylindrical obstacle (in this case, the obstacle is the unit circle
x 2 + y 2 = 1).
(a) Verify that F is irrotational [by definition, F is irrotational if curl(F) = 0].

y
3
2
1
x
3 2 1 1 2 3
1
2
3
x
FIGURE 5 The vector field for (x, y) = x + .
x 2 + y2

(b) Verify that F is tangent to the unit circle at each point along the unit circle except (1, 0) and (1, 0) (where F = 0).
(c) What is the circulation of F around the unit circle?
(d) Calculate the line integral of F along the upper and lower halves of the unit circle separately.
SOLUTION
(a) In Exercise 8, we proved the identity curl( ) = 0. Since F is a gradient vector field, it is irrotational; that is,
curl(F) = 0 for (x, y)  = (0, 0), where F is defined.
(b) We compute F explicitly:
   
y2 x 2 2x y
F = = , = 1+ ,
x y (x 2 + y 2 )
2
(x 2 + y 2 )
2

Now, using x = cos t and y = sin t as a parametrization of the circle, we see that
   
F = 1 + sin2 t cos2 t, 2 cos t sin t = 2 sin2 t, 2 cos t sin t ,

and so

F = 2 sin t sin t, cos t = 2 sin t y, x ,

which is clearly perpendicular to the radial vector x, y for the circle.
(c) We use our expression of F from Part (b):
 
y2 x 2 2x y
F = = 1 + 2
, 2
(x 2 + y 2 ) (x 2 + y 2 )
Now, F is not defined at the origin and therefore we cannot use Greens Theorem to compute the line integral along the
unit circle. We thus compute the integral directly, using the parametrization

c(t) = (cos t, sin t), 0 t 2 .


y

x
1
Chapter Review Exercises 1289

Then,
 
sin2 t cos2 t 2 cos t sin t
F (c(t)) c (t) = 1 + 2
, 2
 sin t, cos t
(cos2 t + sin2 t) (cos2 t + sin2 t)
   
= 1 + sin2 t cos2 t, 2 cos t, sin t  sin t, cos t = 2 sin2 t, 2 cos t sin t  sin t, cos t

= 2 sin3 t 2 cos2 t sin t = 2 sin t (sin2 t + cos2 t) = 2 sin t

Hence,
  2
F ds = 2 sin t dt = 0
C 0

(d) We denote by C1 and C2 the upper and lower halves of the unit circle. Using part (c) we have
   
F ds + F ds = 0 F ds = F ds (1)
C1 C2 C2 C1

C1

x
1

C2

To compute the circulation along C1 , we compute the integral as in part (c), only that the limits of integration are now
t = 0 and t = . Using the computations in part (c) we obtain
 
F ds = 2 sin2 t dt = 4
C1 0

Therefore, by (1),

F ds = 4.
C2

36. Figure 6 shows the vector field F = , where (x, y) = ln(x 2 + (y 1)2 ) + ln(x 2 + (y + 1)2 ), which is the
velocity field for the flow of a fluid with sources of equal strength at (0, 1) (note that is undefined at these two points).
Show that F is both irrotational and incompressible, that is, curlz (F) = 0 and div(F) = 0 [in computing div(F), treat F
as a vector field in R3 with a zero z-component]. Is it necessary to compute curlz (F) to conclude that it is zero?

y (0, 1)

(0, 1)
FIGURE 6

SOLUTION Since F is a gradient field it is irrotational. This property was proved in Exercise 8, where we showed that
curl( ) = 0 for all . To show that F is incompressible, we first find F explicitly.
   
2x 2x 2(y 1) 2(y + 1)
F(x, y, z) = , , = + , + ,0
x y z x 2 + (y 1)2 x 2 + (y + 1)2 x 2 + (y 1)2 x 2 + (y + 1)2
= F1 , F2 , F3 

Hence,
F1 F2 F3
div F(x, y, z) = + +
x y z
1290 C H A P T E R 18 F U N D A M E N TA L TH E O R E M S O F V E C T O R A N A LY S I S (ET CHAPTER 17)

2 x 2 + (y 1)2 2x 2x 2 x 2 + (y + 1)2 2x 2x
= +
x 2 + (y 1)2 x 2 + (y + 1)2

2 x 2 + (y 1)2 2(y 1) 2(y 1) 2 x 2 + (y + 1)2 2 2(y + 1)2
+ +
x 2 + (y 1)2 x 2 + (y + 1)2

=0+0=0

Note that, again by Exercise 8, the divergence of is zero, and hence so also is the divergence of F.
37. In Section 18.1, we showed that if C is a simple closed curve, oriented counterclockwise, then the line integral is

1
Area enclosed by C = x dy y dx 1
2 C
Suppose that C is a path from P to Q that is not closed but has the property that every line through the origin intersects
C in at most one point, as in Figure 7. Let R be the region enclosed by C and the two radial segments joining P and Q to
the origin. Show that the line integral in Eq. (1) is equal to the area of R. Hint: Show that the line integral of F = y, x
along the two radial segments is zero and apply Greens Theorem.
y

C
Q
R
P

x
FIGURE 7

SOLUTION

C
Q
R
P

x
Q

Let F = y, x. Then P = y and Q = x, and Q P


x y = 2. By Greens Theorem, we have
    
y d x + x d y + y d x + x d y + y d x + x d y = 2dA = 2 dA
C QO OP R R
Denoting by A the area of the region R, we obtain
  
1 1 1
A= y d x + x d y + y d x + x d y + y d x + x d y (1)
2 C 2 QO 2 OP
We parametrize the two segments by


Q O : c(t) = (t, t tan ) c (t) = 1, tan

O P : d(t) = (t, t tan ) d (t) = 1, tan 

Then,



F (c(t)) c (t) = t tan , t 1, tan = t tan + t tan = 0
F (d(t)) d (t) = t tan , t 1, tan  = t tan + t tan = 0

Therefore,
 
F ds = F ds = 0.
QO OP

Combining with (1) gives



1
A= y d x + x d y.
2 C
Chapter Review Exercises 1291

38. Suppose that the curve C in Figure 7 has the polar equation r = f ( ).
(a) Show that c( ) = ( f ( ) cos , f ( ) sin ) is a counterclockwise parametrization of C.
(b) In Section 12.4, we showed that the area of the region R is given by the formula

1
Area of R = f ( )2 d
2
Use the result of Exercise 37 to give a new proof of this formula. Hint: Evaluate the line integral in (1) using c( ).
SOLUTION
(a) The curve r = f ( ) in polar coordinates can be parametrized using as a parameter. Since x = r cos and
y = r sin , we have

x = r cos = f ( ) cos , y = r sin = f ( ) sin .

When varies from to , the path C is traversed counterclockwise.


(b) In Exercise 37 we showed that

1
area of R = x dy y dx (1)
2 C
We evaluate the line integral using the parametrization in (a):

c( ) = ( f ( ) cos , f ( ) sin ) , .

We have
dy  
= f  ( ) sin + f ( ) cos d y = f  ( ) sin + f ( ) cos d
d
dx  
= f  ( ) cos f ( ) sin d x = f  ( ) cos f ( ) sin d
d
Hence
 
    
x dy y dx = f ( ) cos f  ( ) sin + f ( ) cos f ( ) sin f  ( ) cos f ( ) sin d
C

= f ( ) f  ( ) cos sin + f 2 ( ) cos2 f ( ) f  ( ) sin cos + f 2 ( ) sin2 d

 
= f 2 ( ) cos2 + sin2 d = f 2 ( ) d

Substituting in (1) we obtain

1 2
area of R = f ( ) d
2

39. Prove the following generalization of Eq. (1). Let C be a simple closed curve in the plane (Figure 8)

S: ax + by + cz + d = 0

Then the area of the region R enclosed by C is equal to



1
(bz cy) d x + (cx az) d y + (ay bx) dz
2n C
where n = a, b, c is the normal to S and C is oriented as the boundary of R (relative to the normal vector n). Hint:
Apply Stokes Theorem to F = bz cy, cx az, ay bx.

Plane S
n = a, b, c

C
R

y
x

FIGURE 8
1292 C H A P T E R 18 F U N D A M E N TA L TH E O R E M S O F V E C T O R A N A LY S I S (ET CHAPTER 17)

SOLUTION By Stokes Theorem,


  
curl(F) dS = (curl(F) en ) d S = F ds (1)
S S C
We compute the curl of F:
 
 i j k 
 
 
 
curl(F) =   = 2ai + 2bj + 2ck = 2 a, b, c
 x y z 
 
 bz cy cx az ay bx 

The unit normal to the plane ax + by + cz + d = 0 is


a, b, c
en = 
a + b2 + c2
2

Therefore,
1
curl(F) en = 2 a, b, c  a, b, c
a + b2 + c2
2

2 
=  (a 2 + b2 + c2 ) = 2 a 2 + b2 + c2
a 2 + b2 + c2
Hence,
     
curl(F) dS = curl(F) en d S = 2 a 2 + b2 + c2 d S = 2 a 2 + b2 + c2 1 dS (2)
S S S S

The sign of S 1 dS is determined by the orientation of S. Since the area is a positive value, we have
  
 
 1 ds  = Area (S)

S

Therefore, (2) gives


   
 
 curl(F) dS = 2 a 2 + b2 + c2 Area(S)

S

Combining with (1) we obtain


  
 
2 a 2 + b2 + c2 Area(S) =  F ds
C
or
 
1 1  
Area(S) =  =  (bz cy) d x + (cx az) d y + (ay bx) dz 
2 a 2 + b2 + c2 2n C

40. Use the result of Exercise 39 to calculate the area of the triangle with vertices (1, 0, 0), (0, 1, 0), and (0, 0, 1) as a
line integral. Verify your result using geometry.
SOLUTION In Exercise 39 we showed that if C is a simple closed curve in the plane ax + by + cz + d = 0, then the
area of the region R enclosed by C is equal to

1
(bz cy) d x + (cx az) d y + (ay bx) dz, n = a, b, c (1)
2n C
z

C = (0, 0, 1)

B = (0, 1, 0)
y
A = (1, 0, 0)
x
Chapter Review Exercises 1293

We use this formula where C is the triangle ABC parametrized counterclockwise. We compute the upward-pointing
normal to the plane of the triangle:

n = AB AC = (j i) (k i) = i + k + j = i + j + k.

We substitute n = 12 + 12 + 12 = 3 and a = b = c = 1 in (1) to obtain:

1
area of R = (z y) d x + (x z) d y + (y x) dz (2)
2 3 C
We parametrized the oriented segments by

AB : c1 (t) = (1 t, t, 0), 0t 1 d x = dt, d y = dt, dz = 0


BC : c2 (t) = (0, 1 t, t), 0t 1 d x = 0, d y = dt, dz = dt
C A : c3 (t) = (t, 0, 1 t), 0t 1 d x = dt, d y = 0, dz = dt

We compute the line integral along each segment separately:


  1  1
(z y) d x + (x z) d y + (y x) dz = (0 t)(dt) + (1 t 0) dt = 1 dt = 1
AB 0 0
  1  1
(z y) d x + (x z) d y + (y x) dz = (0 t)(dt) + (1 t 0) dt = 1 dt = 1
BC 0 0
  1  1
(z y) d x + (x z) d y + (y x) dz = (1 t 0) dt + (0 t)(dt) = 1 dt = 1
CA 0 0

The integral along C is the sum of these three integrals. That is,

(z c) d x + (x z) d y + (y x) dz = 1 + 1 + 1 = 3
C
We combine with (2) to obtain the following area of the triangle:

1 3
area of R = 3 =
2 3 2

We verify this solution geometrically. The triangle spanned by the vectors AB = j i and AC = k i has area
1  1
 1
 AB AC  = i + j + k = 3.
2 2 2
The two answers match.

Potrebbero piacerti anche